Otitis Media

nursing case study on otitis media

Otitis media is a common and frequently encountered ear infection that affects individuals of all ages, particularly young children. This condition involves inflammation and infection of the middle ear, often resulting from viral or bacterial pathogens. Otitis media can lead to various symptoms, including ear pain , fever , hearing difficulties, and fluid buildup behind the eardrum .

This article aims to serve as a comprehensive nursing guide to otitis media, diving into its causes, clinical manifestations, diagnostic methods, medical management, and nursing interventions .

Table of Contents

  • What is Otitis Media? 

Pathophysiology

Statistics and incidences, clinical manifestations, assessment and diagnostic findings, pharmacologic management, surgical management, nursing assessment, nursing diagnoses, nursing care planning and goals, nursing interventions, documentation guidelines, what is otitis media.

nursing case study on otitis media

Otitis media is very common among children.

  • Otitis media is an inflammation of the middle ear without reference to etiology or pathogenesis.
  • It can be classified into many variants based on etiology, duration, symptomatology, and physical findings.

nursing case study on otitis media

In children, developmental alterations of the eustachian tube, an immature immune system, and frequent infections of the upper respiratory mucosa all play major roles in AOM development.

  • The most important factor in middle ear diseases is eustachian tube dysfunction, in which the mucosa at the pharyngeal end of the ET is part of the mucociliary system of the middle ear.
  • The eustachian tube in an infant is shorter and wider than in an older child or adult.
  • The tube is also straighter, thereby allowing nasopharyngeal secretions to enter the middle ear more easily.
  • Interference with the mucosa by edema , tumor , or negative intratympanic pressure facilitates the direct extension of infectious processes from the nasopharynx to the middle ear, causing otitis media.

Otitis media is one of the most common infectious diseases of childhood.

  • Two of every three children have at least one episode of otitis media by the time they are 1 year old.
  • Otitis media accounts for approximately 20 million annual physician visits.
  • Various epidemiologic studies report the prevalence rate of acute otitis media to be 17-20% within the first two years of life.
  • One-third of children experience six or more episodes of otitis media by age 7 years.
  • Peak prevalence of otitis media in both sexes occurs in children aged 6 to 18 months.

A multitude of host, infectious, allergic, and environmental factors contribute to the development of otitis media.

  • Immature immune system. Otitis media is an infectious disease that prospers in an environment of decreased immune defenses.
  • Genetic predisposition. Although familial clustering of otitis media has been demonstrated in studies that examined genetic associations of otitis media, separating genetic factors from environmental influences has been difficult.
  • Anatomic abnormality. Children with anatomic abnormalities of the palate and associated musculature have a higher risk for otitis media.
  • Physiologic dysfunction. Abnormalities in the physiologic function of the ET mucosa increase the risk of bacterial invasion of the middle ear and the resultant otitis media.
  • Bacterial pathogens. The most common bacterial pathogen is Streptococcus pneumoniae, followed by Haemophilus influenzae , and Moraxella catarrhalis .
  • Infant feeding methods. Many studies report that breastfeeding protects infants against otitis media.

Otitis media should be suspected in children with a history of characteristic head-neck and general symptoms.

  • Otalgia . Young children may exhibit signs of otalgia by pulling on the affected ear or ears or pulling on the hair ; otalgia apparently occurs more often when the child is lying down.
  • Otorrhea . Discharge may come from the middle ear through a recently perforated tympanic membrane , or through another perforation.
  • Headache. An older child may complain of a headache.
  • Symptoms of upper respiratory infection. Concurrent or recent symptoms of URI, such as cough , rhinorrhea or sinus congestion is common.
  • Fever. Two-thirds of children with otitis media have a history of fever, although fevers greater than 40°C are uncommon.
  • Irritability. Irritability may be the sole early symptom in a young infant or toddler.

Otitis media may be revealed through the following:

  • Laboratory tests. Laboratory evaluation is usually unnecessary, although many experts recommend a full sepsis workup in infants younger than 12 weeks who present with fever and otitis media.
  • Tympanocentesis. The criterion standard in the diagnosis of otitis media is tympanocentesis to determine middle ear fluid, followed by culture of fluid to identify causative pathogens.

Medical Management

In 2013, the American Academy of Pediatrics (AAP) and the American Academy of Family Practice published updated guidelines on the medical management of otitis media.

  • Antibiotic therapy. Among other recommendations, the guidelines recommended antibiotics for bilateral and unilateral otitis media in children aged at least 6 months with severe signs and symptoms.

The FDA has approved more than a dozen antibiotics to treat otitis media.

  • Antimicrobial agents. These agents remove pathogenic bacteria from the middle ear fluid.

From the beginning, it is essential to integrate surgical management of otitis media with medical treatment.

  • Myringotomy and TT placement. Myringotomy or the incision of the eardrums may be performed to establish drainage and to insert tiny tubes into the tympanic membrane to facilitate drainage.
  • Adenoidectomy. The performance of adenoidectomy to treat patients with otitis media has generated extensive discussion and research, though potential benefits are controversial.

Nursing Management

Most infants and children with otitis media are cared for at home; therefore, a primary responsibility of the nurse is to teach the family caregivers about prevention and the care of the child.

Assessment of a child with otitis media include the following:

  • Physical examination. The infant’s ear is examined with an otoscope by pulling he ear down and back to straighten the ear canal.
  • History. Assess if there is a history of trauma to the ears, affected siblings, a history of cranial/facial defects or any familial history of otitis media.

Based on the assessment data, the major nursing diagnoses are:

  • Acute pain related to the inflammation of the middle ear.
  • Anxiety related to health status.
  • Impaired verbal communication related to effects of hearing loss.
  • Disturbed sensory perception related to obstruction, infection of the middle ear, or auditory nerve damage.
  • Risk for injury related to hearing loss, decreased visual acuity.
  • Infection related to presence of pathogens.

Main Article: 4 Otitis Media Nursing Care Plans 

The major goals for the child with otitis media are:

  • The child or parent will indicate absence of pain .
  • The child will be free of infection.
  • The parents will state understanding of preventive measures.
  • The child will have normal hearing.

Nursing care for the child with otitis media include:

  • Positioning . Have the child sit up, raise head on pillows, or lie on unaffected ear.
  • Heat application. Apply heating pad or a  warm hot water bottle.
  • Diet. Encourage breastfeeding of infants as breastfeeding affords natural immunity to infectious agents; position bole-fed infants upright when feeding .
  • Hygiene . Teach family members to cover mouths and noses when sneezing or coughing and to wash hands frequently.
  • Monitoring hearing loss. Assess hearing ability frequently.

Goals are met as evidenced by:

  • The child or parent indicated absence of pain .
  • The child is free of infection.
  • The parents stated understanding of preventive measures.
  • The child has normal hearing.

Documentation in a child with otitis media include the following:

  • Individual findings, including factors affecting, interactions, nature of social exchanges, specifics of individual behavior.
  • Cultural and religious beliefs, and expectations.
  • Plan of care.
  • Teaching plan.
  • Responses to interventions, teaching, and actions performed.
  • Attainment or progress toward desired outcome.

5 thoughts on “Otitis Media”

This was very useful..thank you

One tip I always share with parents is to make sure their child is up to date on their pneumococcal vaccinations, as pneumococcal bacteria are a common cause of ear infections. Another important tip is to encourage frequent hand washing, especially after being in crowded or public places, to help prevent the spread of infections. It’s also important to monitor for signs of an ear infection, such as pain, redness, or discharge, and to seek prompt medical attention if these symptoms occur. Thank you for bringing attention to this important topic!

I have this and It absolutely sucks! I started getting Eustachian tubes put in when I was 18months old, and still to this day I am having Estuation tubes put in every other year or so. I am currently, 24. I got my first set of hearing aids when I turned 17, bilateral. I hope one day in the future we can have an answer to permanently treat this instead of creating more and more scar tissue on my eardrums every single surgery I get. Scar tissue gets worse and worse every surgery I have. It’s building up on my eardrum, so it’s just decreasing my hearing every single time.

Hello Skyler M,

I’m really sorry to hear about your ongoing struggle with otitis meida and hearing loss. It sounds incredibly tough, especially with the impact on your hearing. It’s totally understandable to hope for a better, more lasting solution.

Have you explored any new treatment options lately? Sometimes specialists in audiology or advanced otology might have different approaches worth considering.

Leave a Comment Cancel reply

Nursing Care Plan (NCP) for Otitis Media / Acute Otitis Media (AOM)

Watch More! Unlock the full videos with a FREE trial

Included In This Lesson

Study tools.

Access More! View the full outline and transcript with a FREE trial

Lesson Objective for Nursing Care Plan (NCP) for Otitis Media/Acute Otitis Media (AOM)

By the end of this lesson, nursing students will be proficient in developing a Nursing Care Plan (NCP) for Otitis Media, specifically Acute Otitis Media (AOM). 

  • Gain a comprehensive understanding of the pathophysiology and etiology of Otitis Media, with a focus on Acute Otitis Media (AOM).
  • Differentiate between AOM and other types of ear infections.
  • Learn how to perform a thorough nursing assessment to identify signs and symptoms of AOM.
  • Understand diagnostic procedures, such as otoscopy and tympanometry, for accurate AOM diagnosis.
  • Explore nursing interventions and management strategies for AOM, including pharmacological and non-pharmacological approaches.
  • Understand the role of pain management and the administration of prescribed medications.
  • Develop effective communication skills to educate patients and caregivers on AOM, its prevention, and treatment.
  • Provide guidance on strategies to prevent recurrent ear infections.
  • Understand the importance of collaboration with other healthcare providers, such as ENT specialists, in managing complicated cases of AOM.
  • Learn when to refer patients for further evaluation and treatment.

Pathophysiology of Otitis Media/Acute Otitis Media (AOM)

  • The pathophysiology of AOM involves an inflammatory response within the middle ear in response to infection. Bacterial or viral pathogens infiltrate the middle ear cavity, leading to localized inflammation.
  • Eustachian tube dysfunction plays a crucial role in AOM. The tube, which normally equalizes pressure and drains fluid from the middle ear, may become blocked or fail to function properly, allowing pathogens to accumulate and multiply.
  • Inflammation of the middle ear mucosa results in the production of fluid. This accumulation of fluid, known as effusion, can lead to impaired hearing and creates an environment conducive to bacterial growth.
  • In bacterial AOM, pathogens like Streptococcus pneumoniae, Haemophilus influenzae, or Moraxella catarrhalis invade the middle ear space. The infection causes additional inflammation and contributes to the signs and symptoms associated with AOM.
  • Viral respiratory infections often precede AOM. Viruses such as respiratory syncytial virus can compromise the mucosal lining of the respiratory tract, making it more susceptible to secondary bacterial infection in the middle ear.
  • As fluid accumulates and pressure within the middle ear increases, individuals with AOM may experience pain and discomfort. The pressure changes can also affect the movement of the tympanic membrane, contributing to hearing impairment.

Etiology of Otitis Media/Acute Otitis Media (AOM)

  • Acute Otitis Media often arises from infections, commonly bacterial (Streptococcus pneumoniae, Haemophilus influenzae, Moraxella catarrhalis) or viral (respiratory syncytial virus, influenza). These infections lead to inflammation and fluid accumulation in the middle ear.
  • Dysfunction of the Eustachian tube, which connects the middle ear to the back of the throat, can contribute to the development of Otitis Media. Impaired tube function may lead to poor ventilation and fluid retention in the middle ear.
  • AOM often follows upper respiratory tract infections, such as the common cold. Pathogens can travel from the upper respiratory system to the middle ear, causing infection and inflammation.
  • Anatomical factors, such as cleft palate or structural abnormalities in the nasopharynx, may increase the risk of Otitis Media by affecting the normal function of the Eustachian tube.
  • Exposure to secondhand smoke and other environmental pollutants can irritate the respiratory system and increase the susceptibility to ear infections, especially in children.
  • Infants and young children are more prone to Otitis Media due to the smaller and more horizontal orientation of the Eustachian tube in early life. This anatomical difference makes drainage less efficient and increases the risk of fluid accumulation.

Desired Outcome of Nursing Care for AOM

  • Minimize pain and discomfort associated with AOM through effective pain management strategies.
  • Achieve resolution of the infection, as evidenced by the absence of signs and symptoms.
  • Prevent complications such as eardrum perforation or the spread of infection to adjacent structures.
  • Ensure understanding of AOM, its treatment, and preventive measures by providing comprehensive education.
  • Enhance the overall quality of life by addressing symptoms promptly and preventing recurrence.

Otitis Media / Acute Otitis Media (AOM) Nursing Care Plan

Subjective data:.

  • Fussiness/irritability 
  • Hearing loss

Objective Data:

  • Tugging/pulling at ear
  • Fluid drainage from ear
  • Lack of balance

Nursing Assessment for Otitis Media/Acute Otitis Media (AOM)

  • Obtain a detailed medical history, including the onset and duration of symptoms, previous ear infections, and any recent upper respiratory infections.
  • Evaluate the intensity and characteristics of ear pain, considering the use of a pain scale appropriate for the patient’s age.
  • Perform an otoscopic examination to visualize the tympanic membrane, looking for signs of inflammation, redness, or bulging.
  • Monitor body temperature to assess for fever, a common symptom of AOM.
  • Evaluate hearing status, as AOM can temporarily affect hearing due to the accumulation of fluid in the middle ear.
  • Note any discharge from the ear, as purulent drainage may indicate a ruptured eardrum.
  • Inquire about balance disturbances, as severe cases of AOM may affect the vestibular system.
  • Observe changes in behavior, especially in pediatric patients, as irritability and changes in sleep patterns may be indicative of AOM.

Implementation of Nursing Care Plan for Otitis Media/Acute Otitis Media AOM

  • Administer analgesics as prescribed and provide comfort measures to alleviate pain.
  • Administer antibiotics as prescribed for bacterial AOM to eliminate the causative pathogen.
  • Apply warm compresses to the affected ear to provide relief and promote drainage.
  • Encourage increased fluid intake to prevent dehydration and facilitate recovery.
  • Advocate for adequate rest and comfort to support the body’s healing process.

Nursing Interventions and Rationales

Assess Vital Signs  pain and fever can increase HR, RR, and BP. (Fever most common symptom)
Observe ears and throat for signs of drainage or discharge  Congestion, post-nasal drip, and drainage of the ears may be present. Co-infections such as strep throat, a cold or the flu may also be present 
Assess pain level  Wong-Baker FACES and FLACC scales may be used to assess pain in young children and infants. Pulling at the ears and tilting the head are also signs of ear pain. Use numeric scale for adults (1-10)
Assess for hearing loss/changes in speech  Sounds may be distorted or muffled in the affected ear. Toddlers learning to talk may have changes in speech due to impaired ability to hear. 
Position patient for comfort; sitting up or lying on side of unaffected ear Lying flat or on the side of the affected ear can cause more swelling and fluid accumulation in the eustachian tube, resulting in increased pain. Encourage the parent to hold infants and young children upright to reduce discomfort 
Give medications (pain, antibiotics) & non-pharmacologic interventions Analgesics such as acetaminophen may be given. Other methods include applying warm (not hot) moist compresses to the ears 

Antibiotics are usually given for bacterial infections. A full 10-day course is generally required (DO NOT stop taking course even if they feel better. Take full therapy as a worsening reoccurrence can happen

Prepare family/patient/caregiver for Tympanostomy Tube Placement tube placement may be needed if a child has 3 ear infections within 6 months or 4 infections a year. These tubes will allow the fluid to drain from the ear and reduce complications of hearing loss, speech delay, spread of infection, and tearing of eardrum
Provide education for parent/caregiver

-F/U care 

-Avoid giving bottles or sippy cups while lying down

Practice good hand hygiene to prevent the spread of bacteria that cause ear infections

-Some infections may be resistant to certain antibiotics. Encourage follow-up after treatment to determine if the infection has cleared, even if symptoms seem to subside or resolve.

 Evaluation of Nursing Care for AOM 

  • Regularly reassess pain levels and evaluate the effectiveness of pain management interventions.
  • Monitor for resolution of infection by assessing the reduction of signs and symptoms.
  • Evaluate the patient’s adherence to antibiotic therapy and address any concerns or side effects.
  • Assess understanding of AOM, treatment, and preventive measures through verbal feedback or demonstration.
  • Monitor for the prevention of complications, such as the absence of eardrum perforation or the spread of infection.
  • https://www.mayoclinic.org/diseases-conditions/ear-infections/symptoms-causes/syc-20351616
  • https://my.clevelandclinic.org/health/diseases/8613-ear-infection-otitis-media

View the FULL Outline

When you start a FREE trial you gain access to the full outline as well as:

  • SIMCLEX (NCLEX Simulator)
  • 6,500+ Practice NCLEX Questions
  • 2,000+ HD Videos
  • 300+ Nursing Cheatsheets

“Would suggest to all nursing students . . . Guaranteed to ease the stress!”

View the FULL Transcript

Nursing care plans.

Jon Haws

How do I write a Nursing Care Plan? Why and how do we even use Nursing Care Plans ? This course is going to expand on that for you and show you the most effective way to write a Nursing Care Plan and how to use Nursing Care Plans in the clinical setting . PLUS, we are going to give you examples of Nursing Care Plans for all the major body systems and some of the most common disease processes. When you complete this course, you will be able to write and implement powerful and effective Nursing Care Plans.

0 – Nursing Care Plans Course Introduction

1 – understanding nursing care plans.

  • 16 Questions

Cardiovascular (Cardiac, CVD) Care Plans

  • 5 Questions
  • 6 Questions
  • 10 Questions
  • 3 Questions
  • 8 Questions
  • 1 Questions
  • 4 Questions

Eyes, Ears, Nose, Throat (EENT) Care Plans

  • 7 Questions

Gastrointestinal (GI) Care Plans

  • 2 Questions

Genitourinary (Renal) (Kidney) (Nephrotic) Care Plans

Hematology (blood, labs), oncology (cancer) & immunology (immunity) care plans, integumentary (skin) care plans, mental health care plans, metabolic & endocrine care plans.

  • 9 Questions

Neurological Care Plans

Obstetrics (ob) & pediatrics (peds) care plans, respiratory care plans.

  • 11 Questions

Sepsis (Septic) & Shock Care Plans

Musculoskeletal and skeletal (osteo) (bones) care plans.

U.S. flag

An official website of the United States government

The .gov means it’s official. Federal government websites often end in .gov or .mil. Before sharing sensitive information, make sure you’re on a federal government site.

The site is secure. The https:// ensures that you are connecting to the official website and that any information you provide is encrypted and transmitted securely.

  • Publications
  • Account settings

Preview improvements coming to the PMC website in October 2024. Learn More or Try it out now .

  • Advanced Search
  • Journal List
  • v.14(8); 2022 Aug

Logo of cureus

Etiology, Diagnosis, Complications, and Management of Acute Otitis Media in Children

Abdullah jamal.

1 Department of Otolaryngology, Zain Hospital, Kuwait, KWT

Abdulla Alsabea

2 Department of Cardiology, Mohammed Bin Khalifa Bin Salman Al Khalifa Specialist Cardiac Centre, Awali, BHR

Mohammad Tarakmeh

3 Department of Otolaryngology, Al-Adan Hospital, Kuwait, KWT

Acute otitis media (AOM) is the most common infectious disease encountered by children under the age of two years and the most common cause of antibiotic use in children in the United States. AOM causes irritability, sleeplessness, decreased appetite, imbalance, and dizziness in patients, especially young children. This assessment was conducted to measure the effectiveness of surgical interventions in treating AOM. We reviewed the present findings regarding the etiology, clinical presentations, diagnosis, treatment, and surgical treatment of complications of AOM. Pain associated with AOM (otalgia) can be severe enough to cause parents to seek treatment for their infants or children. Various suggested measures have been used to treat AOM; antibiotic treatment with amoxicillin is still the treatment of choice for AOM, yet other antibiotics may be used in cases of allergy to penicillin or recent use of amoxicillin. Surgical intervention has been introduced and studied as a diagnostic, therapeutic, and preventive measure for AOM; nevertheless, a few studies have shown that surgical interventions are beneficial in treating and preventing AOM compared to the common practice of using antibiotics. Overdiagnosis of AOM is widespread, leading to injudicious antibiotic use, which contributes to antibiotic resistance. Further management should be determined together with the parent, particularly if observation is the primary intervention.

Introduction and background

Acute otitis media (AOM) is defined as "the rapid onset of signs and symptoms of inflammation in the middle ear" [ 1 ]. Recurrent otitis media occurs when episodes of AOM are repeated on three separate and well-documented occasions in a period of the last six months or four or more occasions in the last 12 months [ 2 ]. A more complicated presentation of otitis media is otitis media with effusion (OME), which is considered chronic otitis media. Chronic otitis media presents with liquid in the middle ear cavity and without the signs and symptoms of acute otitis media [ 1 ]. Tympanostomy tube insertion and myringotomy are the most frequently implemented surgical interventions [ 3 ].

AOM is the most common diagnosis worldwide. The Centers for Disease Control and Prevention (CDC) surveillance data showed that the prevalence of AOM in the United States is increasing (150%). Before the age of two years, 70% of children will have encountered at least one AOM episode. Studies have shown that AOM is the main cause of empiric antibiotic prescription in the United States. Various types of otitis media have been identified depending on the presentation of symptoms and complications.

Otolaryngology is a branch of medicine that deals with the ear, nose, and throat from anatomical, physiological, and pathophysiological aspects to assist in the treatment of AOM, and a thorough understanding of these aspects should be assured [ 4 ].

In this study, we reviewed the current findings regarding the etiology, clinical presentations, diagnosis, treatment, and surgical treatment of complications of AOM.

Anatomy of the ear

The ear is a sensory organ that consists of the outer, middle, and inner ears. The outer ear is responsible for collecting and admitting sound waves. It consists of the auricle and meatus, which are also known as the external tubular auditory canal that ends at the ear and drum (outer side of the tympanic membrane). The inner side of the tympanic membrane forms a cavity within the temporal bone, which is referred to as the middle ear. Auditory ossicles are located in the middle ear, malleus, incus, and stapes. The eustachian tube (ET) is a tube that links the back of the nose to the middle ear, whereas the inner ear is responsible for maintaining balance with the aid of the semicircular canals and vestibule, and conducting hearing through the cochlea [ 5 ].

Acute otitis externa

Acute otitis externa (AOE), also known as swimmer’s ear, is an acute inflammatory reaction that affects the subdermis of the external auditory canal and is caused by bacterial infection. During summer, as people tend to swim and partake more in water sports, the prevalence of AOE increases. Factors such as humidity, warmth, alkaline pH in the external canal, and ear scratching can cause acute otitis externa [ 6 , 7 ]. Pseudomonas aeruginosa and Staphylococcus aureus are the predominant bacterial species isolated from patients with AOE, while other bacterial species are, to a lesser extent, encountered and isolated, namely Proteus mirabilis and Klebsiella pneumonia [ 8 ].

Acute otitis media

Acute otitis media (AOM) is purulent or suppurative otitis media. As its name indicates, AOM is characterized by fluid in the middle ear region. This infection of the middle ear cavity is associated with signs and symptoms of rapidly emerging acute inflammation. Ear pain (otalgia) and fever are the most common signs seen in AOM; these distressing signs and their cause are the most common reasons for children’s visits to the clinic [ 9 ]. The connection between the middle ear, which is air-filled and coated with respiratory mucosa, and the nasopharynx is through the ET, which controls fluid drainage and pressure in the middle ear. The anatomy of the ET is slightly different in infants and children than that in adults; the shorter, steeper angled tube found in infants and children transforms to its adult version when a child reaches the age of seven. The tube is usually closed at rest. Sporadic jaw movement due to yawning and swallowing opens it [ 10 ]. Any anatomical or physiological dysfunction in the ET may cause AOM [ 11 ]. The pathophysiology of AOM is simply due to inflammation in the ET, which prevents fluid drainage from the middle ear cavity, and this fluid retention eventually turns into purulent effusion, which is characteristic of AOM. The middle ear cavity is normally a sterile site. The presence of viral or bacterial pathogens, and that, alongside allergic reactions, is usually the initiation factor for ET inflammation that will eventually develop into a sequela of AOM [ 9 ]. Many risk factors have been associated with recurrent AOM, including male sex and genetic susceptibility. For instanceNative Americans have a very high prevalence of AOM compared to African Americans; additionally, daycare attendance usually increases the chances of infection transmission. Other factors, such as pacifier use, which exacerbates ET dysfunction due to sucking pressure, are also associated with recurrent AOM, as are lack of breastfeeding and passive smoking [ 12 , 13 ]. The incidence peak of AOM appears to be at the age of 3-18 months [ 9 ].

Various etiologies have been associated with AOM, such as the biology of the middle ear cleft, differences in anatomical structures between individuals, nasopharynx cell biology, and variations in the immune response to microbial invasions [ 14 ]. Viral pathogens, bacterial pathogens, and genetics have all been associated with AOM; however, bacterial pathogens are believed to be the main causative agents of AOM.

Various clinical studies have evaluated the role of viral pathogens and viral infections as probable etiologies of AOM. Upper respiratory infections caused by viruses such as the respiratory syncytial virus (RSV), influenza virus, and adenovirus pose a greater risk of AOM than the colonization of the nasopharynx with Streptococcus pneumoniae or Haemophilus influenzae [ 15 ]. Additionally, viral upper respiratory tract infections (URTI) cause inflammation of the ET, negative ear pressure, and displacement of secretions in the nasopharynx containing the URTI's causative viral agents into the middle ear cavity [ 16 ]. One study showed a significant correlation between AOM and laboratory-documented epidemics of respiratory viruses [ 17 ]. Respiratory syncytial virus (RSV) is known for its contribution to bronchiolitis and pneumonia in young patients, and its ability to cause acute respiratory illnesses in almost all age groups. According to the CDC, RSV is a common respiratory virus that usually causes mild cold-like symptoms. Studies have shown that RSV-specific intravenous (IV) immunoglobulin prophylaxis and antiviral treatments are effective in preventing AOM episodes [ 18 ].

Isolation of bacterial pathogens from AOM effusion revealed strains of the following bacteria: Streptococcus pneumoniae, Haemophilus influenzae, Moraxella catarrhalis, Streptococcus pyogenes, Staphylococcus aureus, Viridians streptococci, and Pseudomonas aeruginosa. Streptococcus pneumoniae (pneumococcus), a gram-positive, diplococcic, spherical bacterium, was found to be the most common cause of AOM in all age groups. It is responsible for more than 50% of AOM; thus, penicillin-resistant S. pneumoniae is the main cause of treatment failure and recurrence of AOM [ 19 ]. The most commonly identified serotypes of S. pneumoniae that are associated with AOM are 19F, 23F, 14, 6B, 6A, 19A, and 9V. After S. pneumoniae, non-typable H. influenzae is responsible for approximately 20% of AOM episodes in children aged <6 years, around 50% of which produce ꞵ-lactamases [ 20 , 21 ]. Moraxella catarrhalis has a prevalence rate of approximately 10-15%; however, most of its strains develop resistance due to ꞵ-lactamase production, making it responsible for AOM recurrence [ 22 ]. Group A Streptococcus is common in older children and is mainly isolated in cases of perforated eardrums and mastoiditis [ 23 ].

Genetic Factors

Genetic factors are mainly related to defects in the innate immune response of certain individuals, mainly responses linked to TNFA-863A, TNFA-376G, TNFA-238G, IL10-1082 A, and IL6-174G alleles, making them more susceptible to AOM episodes [ 24 ]. These responses alter the production of cytokines that facilitate an inflammatory response, leading to more frequent episodes of AOM [ 25 ]. Treatment should consider targeting the cytokines responsible for mucin production and mucous secretion, and the genes responsible for their expression [ 26 ]. Various vaccines have been studied as a prophylactic measure to reduce the frequency of recurrent AOM, among which the pneumococcal vaccine (PCV7) was found to be effective in preventing otitis media caused by pneumococcal serotypes [ 27 ].

Clinical presentation

The main symptom in patients presenting with AOE is pain, which may be severe. To distinguish AOE from AOM, the pinna or tragus of the patient is moved, which aggravates the pain in patients with AOE. Pain develops continuously for up to two days [ 28 ]. A small amount of milky white discharge is usually accompanied by edema and redness of the ear canal [ 29 ]. AOM mostly affects infants and children, and the inability of children to verbalize their complaints makes it difficult to know exactly what symptoms they are facing. Infants and children with AOM are usually irritable, sleepless, have poor appetites, and frequently tug their ears due to otalgia. Upper respiratory tract infections are the main triggers of AOM; thus, patients may have rhinorrhea. Fluid accumulation in the middle ear canal may cause a decrease in hearing, and excessive fluid retention may cause balance problems or dizziness [ 30 ].

Complications of AOM requiring surgical treatment 

Well-diagnosed and promptly treated AOM is not usually associated with serious complications; nevertheless, persistent symptoms and treatment failures may increase the risk of hearing loss. Some complications of AOM require immediate surgical interventions, such as rupture of the tympanic membrane, treatment failure or antibiotic resistance, AOM affecting immunocompromised patients, and severe otalgia that does not respond to treatment, or hearing impairment, which may develop into hearing loss. If the hearing loss is prolonged, it may affect the child’s IQ and speaking ability, or develop into recurrent AOM [ 31 ].

Proper and correct diagnosis is critical as it leads to appropriate management. In the case of AOE, diagnosis is made by physical exam. A thorough history and careful clinical examination are needed to assist in the diagnosis of AOM. A history of rapid onset signs of inflammation with effusion in the middle ear is evidence of AOM infection. Pneumatic otoscopy is required to examine the mobility of the tympanic membrane of a patient. If a large volume is seen with a flat tympanogram, the tympanic membrane is perforated. Identifying the presence of fluid in the middle ear differs from identifying sterility; thus, AOM cannot be confirmed by tympanometry; rather, it is only ruled out in cases of a normal tympanogram. Similarly, audiometry, which shows a decrease in hearing, does not contribute to the diagnosis of AOM nor helps to distinguish AOM from OME; it only indicates the presence of middle ear fluid [ 32 ].

Preventive measures 

Many prophylactic measures can be taken to reduce or even avoid AOM [ 33 ]. Although the genetic tendency to develop recurrent AOM cannot be avoided, allergic management can ease and decrease the incidence of AOM episodes [ 34 ]. Pneumococcal vaccines can be used and have been shown to be effective in decreasing the incidence of AOM caused by certain serotypes covered by this vaccine. However, S. pneumoniae serotypes, which are not covered by the pneumococcal vaccine, showed an increase in AOM incidence [ 35 ]. Although the Haemophilus influenzae type B vaccine is available (Hib), this serotype is only minimally associated with AOM episodes; thus, taking the vaccine is of limited use [ 36 ]. Furthermore, the efficacy of the trivalent inactivated influenza vaccine (TIV) and the live-attenuated intranasal influenza vaccine (LAIV) in preventing AOM was studied, and it was found that TIV and LAIV were 30% to 55% effective, respectively, especially during allergy seasons [ 37 ]. Long-term chemotherapy with low doses of antibiotics has been studied and proven beneficial in reducing episodes of recurrent AOM; however, this should be used with great caution because of rapidly emerging antibiotic resistance [ 38 ]. In cases of an infirm ET, tympanostomy or myringotomy tubes may be used. These small tubes can be interpolated into the tympanic membrane while preserving air in the ear cavity for a prolonged period of time, preventing fluid accumulation within the middle ear cavity, and maintaining pressure equalization and balance [ 39 ]. 

Treatment of recurrent AOM is divided into two categories: medical and surgical. Medical therapy is mainly attributed to the use of topical antiseptics and topical and oral antibiotics, and surgical treatment is mainly performed by inserting a tympanostomy tube into the middle ear cavity [ 40 ]. Restoring the slightly acidic media of the outer and middle ear cavities helps resolve ear infections; thus, decreasing the pH by applying acidic solutions is beneficial [ 41 ].

Medical Treatment

Available topical treatments for AOE and AOM with tympanostomy include either a single antimicrobial agent (ciprofloxacin, neomycin, ofloxacin, polymyxin B) or a combination of antimicrobial agents with hydrocortisone available as 2-3% ear drop solutions [ 42 ]. Because these agents are used topically, they are generally effective even in the case of microbial resistance, and the only treatment failure is due to drug delivery failure when the drug either cannot reach the affected area or its contact time is minimal [ 43 ]. Systemic antibiotics are rarely indicated for AOE; they are only used in cases of immunocompromised patients or those with diabetes mellitus. Symptomatic treatment is very important as patients will mainly complain of otalgia; therefore, otalgia, mild to moderate, should be treated first, and for as long as needed with topical or oral analgesics (acetaminophen or NSAIDs), while oral narcotics may be required for severe otalgia [ 44 ]. AAP reviewed the treatment of AOM, and the key statements assured the validity of initial observation and symptomatic treatment for pain management without the use of antimicrobial agents due to the rapidly growing resistance caused by antibiotic misuse, which is dependent on age, the severity of the patient’s case, and ability to access healthcare facilities. If the patient is a child younger than a month, he/she should be treated with antibiotics immediately after the observation period. Cases of moderate-to-severe AOM typically present with severe otalgia and fever exceeding 39°C, which should be treated both symptomatically and with an antimicrobial agent. The use of nasal and oral decongestants, corticosteroids, and antihistamines has shown prolongation of middle ear effusions [ 45 ]. None showed minimization or improvement of healing or complications of AOM [ 46 ]. Amoxicillin is considered the first-line treatment for the management of AOM, and amoxicillin with clavulanic acid is encouraged to broaden antimicrobial coverage for patients with recurrent AOM and recent history of amoxicillin use in the previous month. A dosage of 80 to 90 mg per kg per day is given orally in two divided doses for 10 days or 90:6.4 mg of amoxicillin clavulanate per kg per day is given orally in two divided doses. Cephalosporin is used for patients with penicillin allergies, and macrolides are used if the patient is allergic to penicillin and cephalosporin. If symptoms persist after two to three days of proper antibiotic administration, diagnosis reassessment is considered, and a confirmed diagnosis without symptom improvement suggests treatment failure, which is first aided by an empiric broadening of antibiotic coverage; further treatment failure requires antibiotic-susceptibility as determined by middle ear fluid cultures obtained by tympanocentesis.

Surgical Treatment

The tympanocentesis procedure involves inserting a tiny needle in the tympanic membrane, extending into the middle ear cavity, and aspirating accumulated fluids, thus reducing middle ear cavity pressure. In addition to being diagnostic, tympanocentesis is therapeutically effective in reducing middle ear pain and pressure due to fluid drainage and improves the delivery of antibiotic agents to the infected middle ear cavity. However, it has not shown any positive effect in shortening the duration of effusion or frequency of AOM recurrence [ 47 ]. Recurrent AOM is an indication for tympanostomy tubes, which are used as middle ear pressure-equalizing tubes (PET). Studies have shown the implementation of tympanostomy tubes to significantly reduce mean AOM episodes (1.5x) in a six-month follow-up period [ 48 , 49 ]. Another study compared patients with recurrent AOM taking a placebo with those with tympanostomy tubes and followed them for two years; no difference in episode frequency was observed [ 50 ]. However, tympanostomy tubes improved disease-specific quality of life measures in patients [ 51 ]. Large improvements in the psychosocial realm of physical suffering, hearing loss, speech impairment, emotional distress, and activity limitations were documented in a multicenter, non-randomized study [ 52 , 53 ]. Tympanostomy tube insertion is a small, easy surgical procedure that requires anesthesia, either general or local, and is not associated with major complications such as sensorineural hearing loss, bleeding due to vascular injury, and interruption of the ossicular chain; some common complications may include ear drainage, either ejection or preservation of the inserted tube, benign inflammation formation, and residual perforation. Myringoplasty is required if perforation is combined with secondary infection and hearing loss. The tympanostomy tube may migrate from its location in the ear canal and move behind the intact tympanic membrane in a rare asymptomatic complication [ 54 ]. Performing adenoidectomy alone is not recommended for AOM prevention, and the tympanostomy tube should be considered as it is more beneficial [ 55 ]. Chemoprophylaxis with low doses of amoxicillin showed a significant decrease in AOM frequency compared to placebo in two studies with a follow-up period of one year [ 56 ]. Surgical intervention should be considered in cases of recurrent AOM, either due to ET dysfunction or bacterial resistance, Down’s Syndrome, and craniofacial abnormalities such as cleft palate [ 57 , 58 ].

Formal examination of hearing and the installation of tympanostomy tubes, either alone or in conjunction with adenoidectomy, are options for continued therapy by an otolaryngologist. According to a Cochrane review, the number of patients without AOM in the included trials was higher among those who had ventilation tubes. There is also evidence that the insertion of ventilation tubes improves the quality of life in the short run. Their usefulness in avoiding recurrent AOM is still being contested, especially considering the difficulties associated with the inclusion criteria of randomized controlled studies [ 59 ].

As a result, the clinical practice recommendations of the American Academy of Otolaryngology Head and Neck Surgery currently advocate tympanostomy tubes in children with recurrent AOM and effusion at the time of evaluation [ 60 ].

Complications Associated With Surgical Treatment

Tympanocentesis is considered a safe surgical intervention; however, its efficacy in treating AOM is limited, and it is sometimes only considered a diagnostic measure [ 61 ]. Myringotomy has an associated complication of barotrauma, which is caused by the use of hyperbaric oxygen treatment [ 62 ]. Tympanostomy tube insertion is widely used as a surgical intervention in treating OM, and a set of complications has been associated with this surgery; however, these complications are considered acceptable at their level of incidence and seriousness [ 63 ]. One of the most important complications associated with tympanostomy tube insertion is otorrhea, which involves discharges and effusions of the middle ear cavity that drain out to the external ear. This drainage might occur immediately post-operation or a few weeks later [ 64 ]. Post-operative otorrhea is classified into simple and chronic; simple otorrhea develops mainly immediately post-operation and lasts for no longer than three weeks, responds to topical treatment, and is not repeated. In contrast, chronic otorrhea is persistent or recurrent drainage that may not respond to topical or systemic treatment [ 64 ]. Many children may undergo repeated sessions of tube insertion, and studies have shown that treatment of AOM with tube insertion at a young age (> 2 years) results in a reduced need for repeated tube insertions [ 65 ]. Persistent tympanic membrane perforation is mainly caused by tube extrusion, which requires tympanoplasty for closure [ 66 ]. Cholesteatomas were reported (1.1%) due to retraction of the tympanostomy tube, with a higher incidence in young children (< 5 years), repeated tube insertions, longer tube incubation periods, chronic otorrhea, and the use of certain tympanostomy tubes (Goode T-tube) [ 67 ]. Although surgical interventions are uncommon in cases of AOM, once a patient’s condition requires surgical intervention, it should be evaluated and implemented. Children’s parents should be informed about possible complications of the surgery while emphasizing that surgical intervention is not an ultimate cure for the condition, as patients may develop AOM episodes one-year post-operation [ 68 ]. Apart from complications associated with tympanostomy tube insertion, attention is focused on complications associated with the patient’s preparation for the operation, either with general or local anesthesia. Each has its own adverse events that should be evaluated and communicated to the patient and their parents. Common complications of general anesthesia include pain, nausea and vomiting, tooth decay, sore throat, allergy to anesthetic agents, aspiration pneumonia, headache, back pain, and hypothermia. Serious complications include cardiovascular collapse, respiratory depression, nerve injury, pulmonary embolism, idiosyncratic reactions, and death [ 69 ]. Local anesthesia has fewer serious side effects such as headache, hypotension, bradycardia, direct nerve damage, hypothermia, urinary retention, and pain, as patients may still feel pain despite local anesthesia [ 57 , 58 , 69 ].

Conclusions

Acute otitis media (AOM) is without a doubt one of the most common inflammatory disorders in children. It is a major source of morbidity in children and one of the most prevalent causes of antibiotic prescriptions. However, AOM diagnosis may be problematic because symptoms and indicators are not always conclusive and physical examination can be difficult in this age range. A reddish tympanic membrane alone does not rule out AOM, but a hazy bulging membrane with pneumatic otoscopic characteristics associated with effusion against the backdrop of a typical clinical history is pathognomonic of the condition. Furthermore, AOM management has been highly contested, with various distinct treatment regimens. The adoption of these principles is further confounded by a misunderstanding of the many types of OM. Overdiagnosis of AOM is believed to be widespread, leading to incorrect antibiotic use, which contributes to the development of antibiotic resistance and increases the risk of adverse effects. If observation is chosen as the primary intervention, further management should be determined alongside the parent.

The content published in Cureus is the result of clinical experience and/or research by independent individuals or organizations. Cureus is not responsible for the scientific accuracy or reliability of data or conclusions published herein. All content published within Cureus is intended only for educational, research and reference purposes. Additionally, articles published within Cureus should not be deemed a suitable substitute for the advice of a qualified health care professional. Do not disregard or avoid professional medical advice due to content published within Cureus.

The authors have declared that no competing interests exist.

Otitis Media Nursing Diagnosis & Care Plan

Otitis media involves the inflammation or infection of the middle part of the ear, which is the space located behind the eardrum.

Otitis Media Causes

Signs and symptoms of otitis media.

The following can be noted in a child that is suspected to have otitis media:

Expected Outcomes

Nursing assessment for otitis media.

Physical examination: Examining the ear for signs of inflammation or infection, including redness, swelling, and discharge. Checking for fever and other signs of illness.

Tympanometry: A test that measures the movement of the eardrum in response to changes in air pressure, which can help diagnose the type of otitis media.

Nursing Interventions for Otitis Media

Assess the hearing ability of the patient. Older children can be asked questions if there is muffling or absence of sounds in one ear. Rationale: To establish a baseline assessment of otitis media in terms of hearing capacity.

Administer antibiotics and medications as prescribed. Rationale: Otitis media that is caused by a bacterial infection may need a full course of antibiotics.

Assess the caregiver’s readiness to learn, misconceptions, and blocks to learning (e.g. decreased cognitive ability). Rationale: To address cognition and mental status towards the new diagnosis of otitis media and to help overcome blocks to learning.

Nursing Care Plan for Otitis Media

Disturbed sensory perception, deficient knowledge.

Nursing Diagnosis: Deficient Knowledge related to new diagnosis of otitis media as evidenced by the parent of the child’s verbalization of “I want to know more about how to treat the ear infection of my child.”

More Otitis Media Nursing Diagnoses

Sample nursing test questions for otitis media.

c) Haemophilus influenzae

Rationale: Haemophilus influenzae is the most common bacterial cause of otitis media, accounting for approximately 30-40% of cases. Streptococcus pneumoniae and Moraxella catarrhalis are other common bacterial pathogens associated with otitis media.

b) Severe ear pain

Answer: b) Severe ear pain

a) Administering oral antibiotics as prescribed

d) Administering over-the-counter analgesics

Answer: a) Administering oral antibiotics as prescribed

4. Which of the following complications can occur if otitis media is left untreated?

c) Bronchitis

Rationale: Untreated or recurrent otitis media can lead to temporary or permanent hearing loss, especially in young children. Sinusitis, bronchitis, and conjunctivitis are not direct complications of otitis media.

b) Keeping the child in a highly humidified environment

Answer: d) Ensuring the child receives recommended immunizations

Nursing References

Gulanick, M., & Myers, J. L. (2022).  Nursing care plans: Diagnoses, interventions, & outcomes . St. Louis, MO: Elsevier.

Best Nursing Books and Resources

These are the nursing books and resources that we recommend.

This is an excellent reference for nurses and nursing students. While it is a great resource for writing nursing care plans and nursing diagnoses, it also helps guide the nurse to match the nursing diagnosis to the patient assessment and diagnosis.

All introductory chapters in this updated version of a ground-breaking text have been completely rewritten to give nurses the knowledge they require to appreciate assessment, its relationship to diagnosis and clinical reasoning, and the goal and use of taxonomic organization at the bedside.

Additionally, it distinguishes between nursing and collaborative approaches and highlights QSEN competencies.

Disclaimer:

This information is not intended to be nursing education and should not be used as a substitute for professional diagnosis and treatment.

Leave a Comment Cancel reply

Website maintenance is scheduled for Saturday, September 7, and Sunday, September 8. Short disruptions may occur during these days.

brand logo

HEIDI L. GADDEY, MD, MATTHEW THOMAS WRIGHT, DO, AND TRACY N. NELSON, MD

Am Fam Physician. 2019;100(6):350-356

Author disclosure: No relevant financial affiliations.

Acute otitis media (AOM) is the most common diagnosis in childhood acute sick visits. By three years of age, 50% to 85% of children will have at least one episode of AOM. Symptoms may include ear pain (rubbing, tugging, or holding the ear may be a sign of pain), fever, irritability, otorrhea, anorexia, and sometimes vomiting or lethargy. AOM is diagnosed in symptomatic children with moderate to severe bulging of the tympanic membrane or new-onset otorrhea not caused by acute otitis externa, and in children with mild bulging and either recent-onset ear pain (less than 48 hours) or intense erythema of the tympanic membrane. Treatment includes pain management plus observation or antibiotics, depending on the patient's age, severity of symptoms, and whether the AOM is unilateral or bilateral. When antibiotics are used, high-dose amoxicillin (80 to 90 mg per kg per day in two divided doses) is first-line therapy unless the patient has taken amoxicillin for AOM in the previous 30 days or has concomitant purulent conjunctivitis; amoxicillin/clavulanate is typically used in this case. Cefdinir or azithromycin should be the first-line antibiotic in those with penicillin allergy based on risk of cephalosporin allergy. Tympanostomy tubes should be considered in children with three or more episodes of AOM within six months or four episodes within one year with one episode in the preceding six months. Pneumococcal and influenza vaccines and exclusive breastfeeding until at least six months of age can reduce the risk of AOM.

Acute otitis media (AOM) is commonly diagnosed in children in primary care offices. It is also a leading contributor to antibiotic prescriptions and medical costs in children. 1 This article provides a summary and review of the best, most recent evidence to guide the diagnosis and treatment of AOM.

Practice guideline from the AAP, which is based on consistent evidence from observational studies
, Expert opinion and practice guideline from the AAP, which is based on consistent evidence from observational studies
Practice guideline from the AAP, which is based on consistent evidence from observational studies
, Practice guideline from the AAP, which is based on consistent evidence from observational studies; Cochrane review on antibiotics for acute otitis media in children
Practice guideline from the AAP, which is based on consistent evidence from observational studies
Do not prescribe antibiotics for otitis media in children two to 12 years of age with nonsevere symptoms if the observation option is reasonable.American Academy of Family Physicians

Epidemiology

AOM is the most common diagnosis in childhood acute sick visits, accounting for 13.6 million office visits among children annually. 1

The incidence of AOM peaks between six and 15 months of age. 2

AOM is marginally more common in boys than in girls. 2

By three years of age, 50% to 85% of children experience at least one episode of AOM. However, after 24 months of age, the risk decreases with increasing age. 2 , 3

Risk factors for AOM are shown in Table 1 . 2 , 4 The most common causative bacterial species are Streptococcus pneumoniae , Haemophilus influenzae , and Moraxella catarrhalis . 5 , 6

Age younger than five years
Craniofacial abnormalities
Family history of ear infections
Low birth weight (less than 2.5 kg [5 lb, 8 oz])
Male sex
Premature birth (before 37 weeks of gestation)
Prior ear infections
Recent viral upper respiratory tract infection
White ethnicity
Exposure to tobacco smoke or environmental air pollution
Factors increasing crowded living conditions (e.g., cold seasons, low socioeconomic level, day care/school)
Gastroesophageal reflux
Lack of breastfeeding
Pacifier use after six months of age
Supine bottle feeding (bottle propping)

In addition to ear pain, AOM is commonly associated with fever, irritability, otorrhea, anorexia, and sometimes vomiting or lethargy. 2

The diagnosis is made clinically using common symptoms and findings on examination of the tympanic membrane. 7

AOM should be diagnosed in symptomatic children with moderate to severe bulging of the tympanic membrane ( Figure 1 4 ) or new-onset otorrhea not caused by otitis externa. 7 It can also be diagnosed in children with mild bulging and either recent-onset ear pain (less than 48 hours) or intense erythema of the tympanic membrane. Ear pain can be assumed in fussy, nonverbal children who hold, tug, or rub the ear. 7

nursing case study on otitis media

AOM should not be diagnosed without evidence of middle ear effusion on pneumatic otoscopy or tympanometry. 7

Otitis media with effusion is often misdiagnosed as AOM. Otitis media with effusion can be distinguished on physical examination by a neutral or retracted (not bulging) tympanic membrane with an amber or blue (not white or pale yellow) hue. Air fluid levels, however, may be present in both conditions. 8

Pneumatic otoscopy should be used for the assessment of the tympanic membrane . 4 , 7 Pneumatic otoscopy is up to 94% sensitive and 90% specific for identifying middle ear effusion. 4 , 9

Tympanometry can be used as an adjunct to pneumatic otoscopy. Tympanometry is also 70% to 94% sensitive and 90% specific for identifying middle ear effusion. 4 , 9

Tympanocentesis is the diagnostic standard for identifying the causative bacteria in the middle ear fluid. However, this procedure is impractical in a primary care clinic, and it rarely changes initial management because the common bacterial pathogens are predictable. 5 However, tympanocentesis may be considered to guide treatment in children with ongoing severe symptoms despite treatment with multiple antibiotics by identifying the offending pathogen and its antibiotic susceptibility. 7

Table 2 lists treatment strategies based on age, symptoms, and physical examination findings. 7

AOM with otorrheaAntibiotic therapy
AOM with severe symptoms or if follow-up cannot be guaranteedAntibiotic therapy
Bilateral AOM without otorrheaSix months to two years of age: antibiotic therapy
Two years and older: antibiotic therapy or observation without initial antibiotic treatment
Unilateral AOM without otorrheaAntibiotic therapy or observation without initial antibiotic treatment†

ANTIBIOTIC THERAPY

The resolution rate of AOM in children is 81% without antibiotic treatment vs. 93% with antibiotic treatment. 1 Thus, antibiotics have limited benefits compared with the potential adverse effects, such as rash, vomiting, or diarrhea. 10

Antibiotic treatment of AOM in children does not decrease early pain (before 24 hours), hearing loss at three months, or recurrence within 30 days. 10

Antibiotic treatment has some beneficial effect on pain after 24 hours (up to 12 days), number of tympanic membrane perforations, and contralateral otitis media. 10 Children younger than two years with bilateral otitis media or otitis media with otorrhea benefit most from antibiotics. 10

If antibiotics are used for AOM, high-dose amoxicillin (80 to 90 mg per kg per day in two divided doses) is first-line therapy , unless the child has taken antibiotics for AOM in the previous 30 days, has purulent conjunctivitis, or has a penicillin allergy. 7

Observation for 48 to 72 hours with deferment of antibiotics should be considered in lower-risk children with AOM. 7 , 10

Amoxicillin/clavulanate (Augmentin) should be the initial antibiotic for children who have taken amoxicillin for AOM in the previous 30 days or who have purulent conjunctivitis. 7

Single-dose intramuscular ceftriaxone is as effective as amoxicillin for isolated episodes of AOM. However, ceftriaxone should not be used as a first-line treatment, because there are limited options if treatment fails. 7

Cefdinir or azithromycin (Zithromax) should be the first-line antibiotic in those with penicillin allergy based on risk of cephalosporin allergy.

Antibiotic treatment failure is defined as diagnosis of AOM in the 30 days following treatment initiation or severe symptoms that do not resolve within 48 to 72 hours of treatment initiation and unimproved findings on tympanic membrane examination. 7

Lack of resolution of AOM is likely secondary to beta-lactamase–producing H. influenzae or M. catarrhalis ; amoxicillin/clavulanate should be used for treatment if amoxicillin fails. 7

Duration of therapy is based on patient age and severity of symptoms. A 10-day course of antibiotics is recommended if the child is younger than two years or has severe symptoms. A five- to seven-day course is effective if the child is two years or older and does not have severe symptoms. 7

Figure 2 is an algorithm for the treatment of AOM in children requiring antibiotics. 7

PAIN MANAGEMENT

Pain should be treated as needed in children with AOM . 7 Monotherapy with oral ibuprofen or acetaminophen provides short-term (less than 48 hours) relief of ear pain secondary to AOM. Evidence is insufficient to establish that one of these medications is superior to the other or that combined therapy provides better pain relief. 11 Caregivers should be counseled on appropriate use of these pain medications. 7

Topical anesthetic eardrops and naturopathic eardrops have been found to decrease pain in some small studies, but overall evidence is insufficient to recommend routine use. 12 , 13 They should be avoided if there is any concern for tympanic membrane perforation. 7

There is limited evidence for the benefit of home remedies, such as distraction, external application of heat or cold, and oil drops into the external auditory canal. 7

For children younger than two years, follow-up of AOM can typically occur at the next scheduled wellness visit or three months after completing treatment to ensure resolution of middle ear fluid. For children two years and older without an upcoming visit or children with recurrent AOM, reevaluation within three months of completing treatment should be considered to ensure resolution of middle ear effusion. 14

Susceptibility to AOM is complex and not well understood, and it likely includes a combination of genetic, anatomic, and environmental factors. 2

The previous heptavalent pneumococcal vaccine reduced the relative risk of AOM by 5% to 6% in high-risk children and up to 6% in low-risk children. For AOM caused specifically by pneumococcus, the relative risk reduction was 20% to 25%. It is difficult to assess the effect of the current 13-valent pneumococcal conjugate vaccine (Prevnar 13) on prevention of AOM, but a recent study showed that the extra six serotypes result in an 86% risk reduction of pneumococcal-specific AOM in middle ear fluid compared with the heptavalent vaccine. 16 All children should receive pneumococcal vaccination according to guidelines from the Advisory Committee on Immunization Practices. 17

AOM often follows a viral upper respiratory tract infection. Influenza vaccination leads to a 4% absolute reduction in AOM episodes and a 30% to 55% reduction in AOM during the respiratory illness season. 7 , 18 Children older than six months should receive annual influenza vaccination. 19

Breastfeeding reduces the risk of AOM. Longer duration of breastfeeding provides greater protection for children younger than two years. 20 Exclusive breastfeeding until six months of age reduces the risk by 43%. 7 , 20

Xylitol (chewing gum, lozenges, or syrup) reduces the occurrence of AOM in children attending day care by 25%. Evidence for the benefit of xylitol in children prone to otitis media or in children with an acute respiratory infection is inconclusive. Adverse effects of xylitol include abdominal pain and rash. 21 Xylitol must be used multiple times per day for the entire respiratory illness season to be effective, leading to poor compliance. 22

Evidence is mixed on the benefit of zinc supplementation for the prevention of AOM in healthy children younger than five years. 23

Weak evidence exists for eliminating tobacco smoke exposure, avoiding supine bottle feeding (bottle propping), and reducing or eliminating pacifier use after six months of age in the prevention of AOM and AOM recurrences. However, given the risk of tobacco smoke exposure on overall health, avoidance is recommended. 7 , 24

Recurrent AOM

Referral to an otolaryngologist for possible tympanostomy tube placement should be considered in children with three or more episodes of AOM within six months or four episodes within one year with one episode in the preceding six months. 7

Tympanostomy tubes should not be placed in children with recurrent AOM if no middle ear effusion is noted at the time of otolaryngologist evaluation. 25

Possible long-term sequelae of tympanostomy tubes include structural changes to the tympanic membrane, such as focal atrophy, tympanosclerosis, retraction pockets, and chronic perforation; cholesteatoma; and chronic otorrhea. 26 These risks should be weighed against the risks associated with chronic otitis media with effusion, including decreased academic performance, vestibular problems, behavioral issues, and overall decreased quality of life. 25

Prophylactic antibiotics should not be prescribed to reduce the frequency of AOM episodes in children with recurrent AOM. They have not been shown to be effective and increase rates of microbial resistance. 7

Special Considerations for Infants

Infants eight weeks and younger are at greater risk of severe sequelae from AOM, including sepsis, meningitis, and mastoiditis. 4

Group B streptococci, gram-negative enteric bacteria, and Chlamydia trachomatis are common pathogens found in the middle ear fluid of neonates younger than two weeks, and a full sepsis workup should be completed for any neonate younger than two weeks with fever and apparent AOM. 26 , 27 Antibiotics should be initiated for sepsis as indicated. Amoxicillin is the first-line antibiotic for neonates older than two weeks. 26 , 27

Special Considerations for Adults

Treatment of AOM in adults is largely extrapolated from studies of treatment in children, with amoxicillin as the recommended first-line antibiotic.

There are no data on observation instead of treatment in adults with AOM. Therefore, adults should be treated with antibiotics at initial presentation to prevent complications.

Adults with recurrent AOM (more than two episodes per year) or otitis media with effusion that persists for more than six weeks should be referred to an otolaryngologist to be evaluated for mechanical eustachian tube obstruction. 4

Practical Considerations

To allow visualization of the tympanic membrane, attempts should be made to safely remove cerumen obstruction/impaction using ceruminolytics, irrigation, or manual removal. No one ceruminolytic has been shown to be superior. 28 If visualization remains difficult, cerumen removal should be attempted again the next day or the patient should be referred to an otolaryngologist. 29 Cerumen impaction was covered previously in American Family Physician . 30

Tympanometry can be difficult in young children. If adequate readings cannot be obtained, cerumen should be removed and proper fit of the device tip should be ensured, then tympanometry reattempted. If this fails, visualized insufflation may be attempted to look for movement of the tympanic membrane.

Other causes of ear pain and erythema of the tympanic membranes, including vascular engorgement from crying, viral and hemorrhagic myringitis, and aberrant tympanic membrane vessels, should be considered before diagnosing AOM. 31

Parents should be counseled that fever and ear pain may persist for 48 to 72 hours after initiation of antibiotics. However, parents should seek care immediately if the child is vomiting or has a high fever, headaches, or pain behind the ear.

This article updates previous articles on this topic by Harmes, et al. 4 ; Ramakrishnan, et al. 32 ; and Pichichero . 33

Data Sources: A PubMed search was completed in Clinical Queries using the key terms pediatric, children, acute otitis media, evaluation, treatment, and antibiotic management. We reviewed the updated Agency for Healthcare Research and Quality Evidence Report on the management of acute otitis media, which included a systematic review of the literature through October 2018. Also searched were Essential Evidence Plus, Clinical Evidence, Google Scholar, and the Cochrane database. Reference lists of retrieved articles were also searched. Search dates: September to October 2018 and May 2019.

The views expressed in this material are those of the authors and do not reflect the official policy or position of the U.S. government, Department of Defense, or Department of the Air Force.

Coco AS. Cost-effectiveness analysis of treatment options for acute otitis media. Ann Fam Med. 2007;5(1):29-38.

Qureishi A, Lee Y, Belfield K, et al. Update on otitis media - prevention and treatment. Infect Drug Resist. 2014;7:15-24.

Klein JO. Epidemiology of otitis media. Pediatr Infect Dis J. 1989;8(1 suppl):S9.

Harmes KM, Blackwood RA, Burrows HL, et al. Otitis media: diagnosis and treatment [published correction appears in Am Fam Physician . 2014;89(5):318]. Am Fam Physician. 2013;88(7):435-440. Accessed July 17, 2019. https://www.aafp.org/afp/2013/1001/p435.html

Pumarola F, Marès J, Losada I, et al. Microbiology of bacteria causing recurrent acute otitis media (AOM) and AOM treatment failure in young children in Spain: shifting pathogens in the post-pneumococcal conjugate vaccination era. Int J Pediatr Otorhinolaryngol. 2013;77(8):1231-1236.

Chen YJ, Hsieh YC, Huang YC, et al. Clinical manifestations and microbiology of acute otitis media with spontaneous otorrhea in children. J Microbiol Immunol Infect. 2013;46(5):382-388.

Lieberthal AS, Carroll AE, Chonmaitree T, et al. The diagnosis and management of acute otitis media [published correction appears in Pediatrics . 2014;133(2):346]. Pediatrics. 2013;131(3):e964-e999.

Shaikh N, Hoberman A, Rockette HE, et al. Development of an algorithm for the diagnosis of otitis media. Acad Pediatr. 2012;12(3):214-218.

Rovers MM, Schilder AG, Zielhuis GA, et al. Otitis media [published correction appears in Lancet . 2004;363(9414):1080]. Lancet. 2004;363(9407):465-473.

Venekamp RP, Sanders SL, Glasziou PP, et al. Antibiotics for acute otitis media in children. Cochrane Database Syst Rev. 2015(6):CD000219.

Sjoukes A, Venekamp RP, van de Pol AC, et al. Paracetamol (acetaminophen) or non-steroidal anti-inflammatory drugs, alone or combined, for pain relief in acute otitis media in children. Cochrane Database Syst Rev. 2016(12):CD011534.

Bolt P, Barnett P, Babl FE, et al. Topical lignocaine for pain relief in acute otitis media: results of a double-blind placebo-controlled randomised trial. Arch Dis Child. 2008;93(1):40-44.

Foxlee R, Johansson A, Wejfalk J, et al. Topical analgesia for acute otitis media. Cochrane Database Syst Rev. 2006(3):CD005657.

Rosenfeld RM, Shin JJ, Schwartz SR, et al. Clinical practice guideline: otitis media with effusion executive summary (update). Otolaryngol Head Neck Surg. 2016;154(2):201-214.

Fortanier AC, Venekamp RP, Boonacker CW, et al. Pneumococcal conjugate vaccines for preventing acute otitis media in children. Cochrane Database Syst Rev. 2019(5):CD001480.

Pichichero M, Kaur R, Scott DA, et al. Effectiveness of 13-valent pneumococcal conjugate vaccination for protection against acute otitis media caused by Streptococcus pneumoniae in healthy young children: a prospective observational study. Lancet Child Adolesc Health. 2018;2(8):561-568.

Centers for Disease Control and Prevention. Pneumococcal vaccine recommendations. Accessed May 15, 2019. https://www.cdc.gov/vaccines/vpd/pneumo/hcp/recommendations.html

Norhayati MN, Ho JJ, Azman MY. Influenza vaccines for preventing acute otitis media in infants and children. Cochrane Database Syst Rev. 2017(10):CD010089.

Centers for Disease Control and Prevention. Flu (influenza). Accessed May 15, 2019. https://www.cdc.gov/vaccines/vpd/flu/index.html

Bowatte G, Tham R, Allen KJ, et al. Breastfeeding and childhood acute otitis media: a systematic review and meta-analysis. Acta Paediatr. 2015;104(467):85-95.

Azarpazhooh A, Lawrence HP, Shah PS. Xylitol for preventing acute otitis media in children up to 12 years of age. Cochrane Database Syst Rev. 2016(8):CD007095.

Danhauer JL, Johnson CE, Baker JA, et al. Will parents participate in and comply with programs and regimens using xylitol for preventing acute otitis media in their children?. Lang Speech Hear Serv Sch. 2015;46(2):127-140.

Gulani A, Sachdev HS. Zinc supplements for preventing otitis media. Cochrane Database Syst Rev. 2014(6):CD006639.

Salah M, Abdel-Aziz M, Al-Farok A, et al. Recurrent acute otitis media in infants: analysis of risk factors. Int J Pediatr Otorhinolaryngol. 2013;77(10):1665-1669.

Rosenfeld RM, Schwartz SR, Pynnonen MA, et al. Clinical practice guideline: tympanostomy tubes in children. Otolaryngol Head Neck Surg. 2013;149(1 suppl):S1-S35.

Nozicka CA, Hanly JG, Beste DJ, et al. Otitis media in infants aged 0–8 weeks: frequency of associated serious bacterial disease. Pediatr Emerg Care. 1999;15(4):252-254.

Turner D, Leibovitz E, Aran A, et al. Acute otitis media in infants younger than two months of age: microbiology, clinical presentation and therapeutic approach. Pediatr Infect Dis J. 2002;21(7):669-674.

Michaudet C, Malaty J. Cerumen impaction: diagnosis and management. Am Fam Physician. 2018;98(8):525-529. Accessed July 17, 2019. https://www.aafp.org/afp/2018/1015/p525.html

Rettig E, Tunkel DE. Contemporary concepts in management of acute otitis media in children. Otolaryngol Clin North Am. 2014;47(5):651-672.

Michaudet C, Malaty J. Cerumen impaction: diagnosis and management. Am Fam Physician. 2018;98(8):525-529. Accessed June 18, 2019. https://www.aafp.org/afp/2018/1015/p525.html

Earwood JS, Rogers TS, Rathjen NA. Ear pain: diagnosing common and uncommon causes. Am Fam Physician. 2018;97(1):20-27. Accessed July 17, 2019. https://www.aafp.org/afp/2018/0101/p20.html

Ramakrishnan K, Sparks RA, Berryhill WE. Diagnosis and treatment of otitis media [published correction appears in Am Fam Physician . 2008;78(1):30]. Am Fam Physician. 2007;76(11):1650-1658. Accessed July 18, 2019. https://www.aafp.org/afp/2007/1201/p1650.html

Pichichero ME. Acute otitis media: part I. Improving diagnostic accuracy. Am Fam Physician. 2000;61(7):2051-2056. Accessed July 18, 2019. https://www.aafp.org/afp/2000/0401/p2051.html

Continue Reading

nursing case study on otitis media

More in AFP

More in pubmed.

Copyright © 2019 by the American Academy of Family Physicians.

This content is owned by the AAFP. A person viewing it online may make one printout of the material and may use that printout only for his or her personal, non-commercial reference. This material may not otherwise be downloaded, copied, printed, stored, transmitted or reproduced in any medium, whether now known or later invented, except as authorized in writing by the AAFP.  See permissions  for copyright questions and/or permission requests.

Copyright © 2024 American Academy of Family Physicians. All Rights Reserved.

  • Remote Access
  • Save figures into PowerPoint
  • Download tables as PDFs

Infectious Diseases: A Case Study Approach

2:  Acute Otitis Media

Aimee Dassner; Jennifer E. Girotto

  • Download Chapter PDF

Disclaimer: These citations have been automatically generated based on the information we have and it may not be 100% accurate. Please consult the latest official manual style if you have any questions regarding the format accuracy.

Download citation file:

  • Search Book

Jump to a Section

Patient presentation.

  • Full Chapter
  • Supplementary Content

Chief Complaint

“Increased irritability and right ear pain.”

History of Present Illness

JL is a 22-month-old female who presents to her primary care provider (PCP) with a 2-day history of rhinorrhea and a 1-day history of increased irritability, fever (to 101.5°F per Mom), and right-ear tugging. Mom denies that JL has had any nausea, vomiting, or diarrhea.

Past Medical History

Full-term birth via spontaneous vaginal delivery. Hospitalized at 9 months of age for respiratory syncytial virus–associated bronchiolitis. Two episodes of acute otitis media (AOM), with last episode about 6 months earlier.

Surgical History

Social history.

Lives with mother, father, and her 5-year-old brother who attends kindergarten. JL attends daycare 2 d/wk, and stays at home with maternal grandmother 3 d/wk.

No known drug allergies

Immunizations

Hepatitis B

Birth

DTap/Hep B/IPV

2 mo, 4 mo, 6 mo

Hib

2 mo, 4 mo, 6 mo, 15 mo

PCV13

2 mo, 4 mo, 6 mo, 12 mo

Influenza

6 mo, 8 mo, 18 mo

MMR

12 mo

Varicella

12 mo

Home Medications

Vitamin D drops 600 IU/d

Physical Examination

Vital signs (while crying).

Temp 100.7°F, P 140 bpm, RR 35, BP 100/57 mm Hg, Ht 81 cm, Wt 23.7 kg

Fussy, but consolable by Mom; well-appearing

Normocephalic, atraumatic, moist mucous membranes, normal conjunctiva, clear rhinorrhea, moderate bulging and erythema of right tympanic membrane with middle-ear effusion

Good air movement throughout, clear breath sounds bilaterally

Cardiovascular

Normal rate and rhythm, no murmur, rub or gallop

Soft, non-distended, non-tender, active bowel sounds

Genitourinary

Normal female genitalia, no dysuria or hematuria

Alert and appropriate for age

Extremities

1. Which of the following clinical criteria is not part of the diagnostic evaluation or staging of acute otitis media (AOM) for this patient?

A. Rhinorrhea

D. Contour of the tympanic membrane

Get Free Access Through Your Institution

Pop-up div successfully displayed.

This div only appears when the trigger link is hovered over. Otherwise it is hidden from view.

Please Wait

nursing case study on otitis media

Case Challenge: Acute Otitis Media in Children--Best Management Strategies

Gordon h. sun, md, ms.

March 28, 2018

Toddler Ear Tugging

A 2-year-old girl was seen by a pediatric nurse practitioner in the ambulatory clinic. The girl's mother reported that her daughter had been fussing and pulling on her right ear for the past 2 days but had no fevers or lethargy. There were no recent sick contacts. The patient had no history of ear surgery. She was not taking any medications and had no known drug allergies.

The patient demonstrated normal vital signs. She appeared nontoxic and was breathing comfortably. She was awake, active, and tugging on her right ear. There was no erythema or edema of the right auricle or mastoid. Otoscopic exam of the right ear demonstrated a normal-appearing external auditory canal and an inflamed and bulging tympanic membrane. The tympanic membrane was immobile on pneumatic insufflation . No otorrhea was seen in the canal. The remainder of the physical exam, including left ear otoscopy, was unremarkable.

nursing case study on otitis media

Medscape Family Medicine © 2018 WebMD, LLC

Any views expressed above are the author's own and do not necessarily reflect the views of WebMD or Medscape.

Cite this: Case Challenge: Acute Otitis Media in Children--Best Management Strategies -  Medscape  - Mar 28, 2018.

Authors and Disclosures

Medical Director of Inpatient Services; Chief, Division of Otolaryngology and Ophthalmology, Rancho Los Amigos National Rehabilitation Center, Downey, California; Adjunct Clinical Associate Professor of Otolaryngology, University of Southern California, Los Angeles, California Disclosure: Gordon H. Sun, MD, MS, has disclosed the following relevant financial relationship: Serve(d) as a director, officer, partner, employee, advisor, consultant, or trustee for: Partnership for Health Analytic Research, LLC

You have already selected for My Alerts

  • Add Other Topics

Click the topic below to receive emails when new articles are available.

You've successfully added to your alerts. You will receive email when new content is published.

  • Perspective
  • Drugs & Diseases
  • Global Coverage
  • Additional Resources

Amoxicillin Shortage: Practical Guidance for Prescribers

  • Social Media a 'Powerful' Way to Talk to Teens About Obesity
  • Prescription for Caution: Can Social Media Posts Jeopardize Your Next Job?
  • From Scrubs to Social Media: How Some Med Students Become Influencers

Otitis Media

  • 2001/viewarticle/confronting-healthcare-disinformation-social-media-2024a1000clb news Confronting Healthcare Disinformation on Social Media
  • 2001/viewarticle/sleep-and-social-media-impact-youth-brain-development-2024a1000ag1 news Sleep and Social Media Impact Youth Brain Development
  • 2001/viewarticle/prioritize-value-social-media-posts-dermatologist-advises-2024a1000f7w news Prioritize Value in Social Media Posts, Dermatologist Advises
  • 2001/viewarticle/surgeon-generals-push-social-media-warning-label-explained-2024a1000c3g news The Surgeon General's Push for Social Media Warning Label Explained
  • Otitis Media
  • Otitis Media With Effusion
  • Acute Otitis Media
  • Complications of Otitis Media
  • Chronic Suppurative Otitis Media
  • Emergent Management of Acute Otitis Media

Classroom Contagions: Get Back-to-School Ready

  • Confronting Healthcare Disinformation on Social Media
  • Sleep and Social Media Impact Youth Brain Development
  • Prioritize Value in Social Media Posts, Dermatologist Advises
  • Managing Common Ear Complaints: An ENT's Advice
  • Bell's Palsy: What to Know in Primary Care
  • Erosive Esophagitis: 5 Things to Know

Acute Otitis Media in Young Children Today

Nursing Geeks

Acute otitis media case study

Acute otitis media case study Case Study 1: HPI: A mother presents with her 2-year-old African American male child with complaints of ear pain and decreased sleep. Earlier this week, he had a runny nose and congestion with a mild cough that occurred mostly when lying down. His temperature is 100.7. PE: Pt is walking around room, exploring in no acute distress. HEENT: Mild nasal congestion, clear postnasal drainage, and lungs clear to auscultation. Ear exam reveals right tympanic membrane erythematous, translucent, in a neutral position, with no pus or fluid noted. Left tympanic membrane is full, reddish/orange in appearance, and opaque with pus. Acute otitis media case study. Neck: Shotty anterior cervical adenopathy Lungs: Clear to auscultation ORDER A PLAGIARISM-FREE PAPER HERE Although many pediatric patients present with common HEENT disorders such as ear infections, allergies, and strep throat, some patients present with rare disorders requiring specialist care. In your role, making this distinction between when to treat and when to refer is essential. For this Discussion, examine the following case studies and consider potential diagnoses and management strategies.   Diagnosis/Differential Diagnosis Based on the signs and symptoms that the patient in this case scenario presented, it is likely that it is an infection of the middle ear called acute otitis media. (Peterson & Reintjes, 2016). These symptoms often occur during or after cold symptoms and are associated with the symptoms of headache, fever otorrhoea, irritability, vomiting, and diarrhea. Similarly, in this case, the child presented with complaints of nasal congestion, a running nose, and a cough before the ear pain Acute otitis media case study. Besides, the clinician visualized a red, translucent, neutrally positioned and non-pus filled right tympanic membrane. The left tympanic membrane was reddish-orange, opaque, full and filled with pus. According to Peterson & Reintjes (2016), visual findings are consistent with acute otitis media. In the advanced form, acute otitis media can precede otitis media with effusion when patients present with vertigo, ringing in the ears or hearing loss. Potential differential diagnoses, in this case, include AOE and OME. In the former, patients present with a red, painful and itchy ear canal Acute otitis media case study. Treatment and Management/Education According to Hochman & Shapiro (2016), amoxicillin is the most recommended antibiotic for managing acute otitis media. However, not all cases of acute otitis media are managed using drugs. In other instances, patients are given supportive treatment since it is a self-limiting illness and this helps to prevent bacterial resistance. Otitis media is preventable using xylitol (Azarpazhooh, Lawrence & Shah, 2016).  This substitute helps to reduce the proliferation and adherence of H influenzae and S. pneumonia in the oral and nasal cavities. Caregivers should be informed of the need to receive influenza and pneumococcal immunizations, avoiding exposure to tobacco and breastfeeding to boost a child’s immunity. Acute otitis media case study. References Azarpazhooh, A., Lawrence, H. P., & Shah, P. S. (2016). Xylitol for preventing acute otitis media in children up to 12 years of age. Cochrane Database of Systematic Reviews. doi:10.1002/14651858.cd007095.pub3 Hochman, M., & Shapiro, N. (2016). Treatment of Acute Otitis Media in Children. 50 Studies Every Pediatrician Should Know, 87-92. doi:10.1093/med/9780190204037.003.0013 Peterson, S., & Reintjes, S. (2016). Otitis Externa, Otitis Media, and Mastoiditis. Oxford Medicine Online. doi:10.1093/med/9780199976805.003.0011 Acute otitis media case study

Is this the question you were looking for? If so, place your order here to get started!

nursing case study on otitis media

Share this:

  • Click to share on Twitter (Opens in new window)
  • Click to share on Facebook (Opens in new window)

Related posts

Discuss art theory of different styles and information about american landscape artists of the 20th century..

Please correct the mistakes I have and add some more information on the topic. The text and information there is quite brief and all over the place. I’d appreciate some more specific information on art theory of different styles...

Ex-health chief underscores needs to strengthen primitive Ph Healthcare system.Discuss

Ex-health chief underscores needs to strengthen primitive Ph Healthcare system. Is this the question you were looking for? If so, place your order here to get started!

Explain how the quality improvement initiative will be evaluated to determine whether there was improvement.

Identify a quality improvement opportunity in your organization or practice. In a 1,250-1,500 word paper, describe the problem or issue and propose a quality improvement initiative based on evidence-based practice. Apply “The Road to Evidence-Based Practice” process, 1. ask...

Case Study: Otitis Media


Otitis Media

Case Presentation

JoAnn was at work when she received a call from the day care where Michael, her five month-old son, was being watched. The child care provider told JoAnn that Michael had been fussy that morning, napped only a short while, refused his bottle, and was running a temperature of 101°F. JoAnn told the day care provider that she would be right over to pick up Michael. She called her pediatrician’s office and scheduled an appointment for that afternoon.

The pediatrician inspected Michael's ears and informed JoAnn that Michael had a middle ear infection. She asked JoAnn if Michael had been coughing a lot and if he recently had a cold or runny nose. She also asked if Michael was breast-fed or bottle-fed and if there was anyone in their household who smoked. JoAnn told the doctor that Michael's nose had been draining quite a bit lately, that he was bottle-fed, and that neither she nor her husband smoked. The doctor wrote a prescription for antibiotics and instructed JoAnn to give Michael a non-prescription childrens' pain medication. An appointment was made for a recheck in two weeks.

Case Background

A middle ear infection, or otitis media, is most often of bacterial origin and commonly follows an upper respiratory infection. The bacteria usually enter the middle ear via the surface of the auditory tube mucus membrane. Inflammation of the tissues in the middle ear results from the infection. The auditory tube becomes swollen or clogged, and pus accumulates in the tympanic cavity of the middle ear as white blood cells rush to the site.

Otitis media can affect anyone, but is most common in young children, with 75% of children experiencing at least one episode by their third birthday. Children are more likely to suffer from otitis media than adults because their immune systems are immature, and their auditory tubes are shorter and straighter than those of adults. Children that attend large day cares, are bottle-fed, and are exposed to cigarette smoke are more likely to experience otitis media.

Symptoms in young children include severe earache, fever, nausea, vomiting, and diarrhea. Rupturing of the tympanic membrane, or eardrum, can also occur but is uncommon. If the condition is very persistent, generally seen as lasting for three months, an operation called a myringotomy can be performed. This operation involves the insertion of a ventilation tube in the tympanic membrane of an infected ear.

Describe the following middle ear structures, explain their functions, and explain how those functions may be impeded by otitis media.

1
2
3
4
5
Date:
My name:
Section ID:
E-mail address:Format:
Me:
My Instructor:
My TA:
Other:
To learn more about the book this website supports, please visit its .
Copyright
Any use is subject to the and |
You must be a registered user to view the in this website.

If you already have a username and password, enter it below. If your textbook came with a card and this is your first visit to this site, you can to register.
Username:
Password:
'); document.write(''); } // -->
( )
.'); } else{ document.write('This form changes settings for this website only.'); } //-->
Send mail as:
'); } else { document.write(' '); } } else { document.write(' '); } // -->
'); } else { document.write(' '); } } else { document.write(' '); } document.write('
TA email: '); } else { document.write(' '); } } else { document.write(' '); } // -->
Other email: '); } else { document.write(' '); } } else { document.write(' '); } // -->
"Floating" navigation? '); } else if (floatNav == 2) { document.write(' '); } else { document.write(' '); } // -->
Drawer speed: '; theseOptions += (glideSpeed == 1) ? ' ' : ' ' ; theseOptions += (glideSpeed == 2) ? ' ' : ' ' ; theseOptions += (glideSpeed == 3) ? ' ' : ' ' ; theseOptions += (glideSpeed == 4) ? ' ' : ' ' ; theseOptions += (glideSpeed == 5) ? ' ' : ' ' ; theseOptions += (glideSpeed == 6) ? ' ' : ' ' ; document.write(theseOptions); // -->
1. (optional) Enter a note here:

2. (optional) Select some text on the page (or do this before you open the "Notes" drawer).
3.Highlighter Color:
4.
Search for:
Search in:
Course-wide Content






News, Articles & Links

Quizzes




More Resources





Instructor Resources





Course-wide Content






  • Practice Test
  • Fundamentals of Nursing
  • Anatomy and Physiology
  • Medical and Surgical Nursing
  • Perioperative Nursing
  • Psychiatric Mental Health Nursing
  • Maternal & Child Nursing
  • Community Health Nursing

Pathophysiology

  • Nursing Research
  • Study Guide and Strategies
  • Nursing Videos
  • Work for Us!
  • Privacy Policy

Rnspeakcom your ultimate guide in nursing

Chronic Suppurative Otitis Media Case Study

otitismediacasestudy

Chronic suppurative otitis media (CSOM) is caused by recurrent ear infections resulting in tympanic membrane perforation characterized by persistent foul-smelling otorrhea from the middle ear. Misconception on ear infection as related to poor hygiene and embarrassing foul-smelling ear discharges generates an erroneous notion of social stigma—which is one of the major causes of hearing loss in CSOM, since most of the cases are not reported and not diagnose earlier without immediate and appropriate treatment. Hearing loss is preventable if only immediate treatment is taken (Smeltzer & Bare, 2004; Acuin, 2004; Bonin et al., 2003).

In the report of Acuin (2004), prevalence surveys confirmed that the global burden of illness from CSOM affects 65–330 million individuals with draining ears, 60% of whom (39–200 million) suffer from significant hearing impairment. CSOM accounts for 28, 000 deaths and a disease burden of over 2 million DALYs. Over 90% of the burden is borne by countries in the South-east Asia and Western Pacific regions, Africa and several ethnic minorities in the Pacific rim. Mastoidectomy with or without tympanoplasty eradicates mastoid infection in about 80% of patients and may be combined with surgical drainage of otogenic abscesses. However, such treatment is costly and does not always lead to satisfactory hearing improvement, and is inaccessible in many developing countries.

Smeltzer & Bare (2004), defines chronic suppurative otitis media as the result of repeated episodes of acute otitis media causing irreversible tissue pathology and persistent perforation of the tympanic membrane. Chronic infections of the middle ear damage the tympanic membrane, destroy the ossicles, and involve the mastoid (Smeltzer & Bare, 2004, Acuin, 2004). CSOM may develop in elderly people who have had an asymptomatic or exacerbated chronic ear infection after URI or when water enters the middle ear during bathing or swimming. Chronic suppurative otitis media may be a complication of a retracted or perforated tympanic membrane. In elderly patients with otorrhea, squamous, and basal cell carcinomas should be considered (Acuin, 2004). Tympanic membrane perforation may be permanent, and the middle ear mucosa may become hypertrophic. In some patients, the inflamed, polypoid hypertrophic mucosa protrudes through the perforation into the external auditory canal as an aural polyp. A cholesteatoma may be present. Some cases are complicated by abscess formation or erosion into bone (Smeltzer & Bare, 2004).

Anatomy and Physiology of Sense of Hearing 

The ear has external , middle and inner ear sections and provides the senses of hearing and equilibrium.

External Ear

The external ear consists of the auricle which collects the sound which then travels down the external auditory meatus .

The middle ear begins with the tympanic membrane , and is an air-filled space ( tympanic cavity ) housing the auditory ossicles . Three auditory ossicles are the malleus, incus and stapes. The tympanic membrane vibrates the malleus, which vibrates the incus, then the stapes. The stapes vibrates the fluid inside the oval window of the inner ear. Auditory ossicles both transmit and amplify sound waves.

Auditory Tube

The auditory ( eustachian ) tube connects the middle ear to the throat to help maintain equal air pressure on both sides of the eardrum .

The inner ear is made up of a membranouslabyrinth inside an osseous labyrinth . Between the two labyrinths is a fluid called perilymph . Endolymph is inside the membranous labyrinth. The cochlea houses the organ of hearing while semicircular canals function in equilibrium. Within the cochlea, the oval window leads to the upper compartment, called the scala vestibuli . A lower compartment, the scala tympani , leads to the round window . The cochlear duct lies between these two compartments and is separated from the scala vestibule by the vestibular membrane , and from the scala tympani by the basilar membrane . The organ of Corti , with its receptors called hair cells, lies on the basilar membrane . Hair cells possess hairs that extend into the endolymph of the cochlear duct . Above the hair cells lies the tectorial membrane , which touches the tips of the stereocilia . Vibrations in the fluid of the inner ear cause the hair cells to bend resulting in an influx of calcium ions. This causes the release of a neurotransmitter from vesicles that stimulate the ends of nearby sensory neurons.

Auditory Nerve Pathways

Nerve fibers carry impulses to the auditory cortices of the temporal lobes where they are interpreted.

Diagnostic Examination

Cbc with wbc differential count.

Hemoglobin, hematocrit, and red blood cell counts may reveal normal. WBC differential count reveals elevated white blood cells, neutrophils, lymphocytes and monocytes counts that indicate severe inflammatory process and bacterial infections (McFarland & Grant, 1988).

Culture and Sensitivity Testing of discharge reveal the presence of staphylococcus aureus – one of the most common bacterial pathogens of otitis media (Smeltzer & Bare, 2004).

Staphylococcus aureus is a respiratory pathogen that may have been insufflated from the nasopharynx into the middle ear through the Eustachian tube during bouts of upper respiratory infections. The bacteria is infrequently found in the skin of the external canal, but may proliferate in the presence of trauma, inflammation, lacerations, or high humidity which may then gain entry to the middle ear through a chronic perforation (Acuin, 2004).

CT Scan revealed perforation of the tympanic membrane, ossicular abscess, and erosions of the bony partitions of the mastoid air cells. Absence or presence of cholesteatoma. Labyrinth and temporal bones may be intact or damaged.

The staphylococcus areus has infected and eroded the tympanic membrane causing perforation, ossicles and the mastoid bony partitions which may cause a conductive hearing loss. CSOM can cause chronic mastoiditis and lead to cholesteatoma formation. It can occur in the middle ear, mastoid cavity, or both. If untreated, cholesteatoma will continue to enlarge, possibly causing damage to the facial nerve and horizontal canal and destruction of other surrounding structures. The absence of cholesteatoma is a good indication that the condition is not malignant and not cancerous. Intactness of the labyrinth and temporal bones are signs that there will be no sensorineural defect (Smeltzer & Bare, 2004; Acuin, 2004).

Chronic Suppurative Otitis Media Assessment Tool and Treatment Model

Nursing care plan.

            The two major problems of chronic suppurative otitis media are infection and pain (particularly, in the presence of mastoiditis). In case of fever, infection is the underlying cause of fever—if the infection is treated, then fever will disappear.

(Doenges, M. E. et al, 2008; Gulanick, M. & Myers, J. L., 2007; Johnson, M. et al , 2007; Roth, R. A., 2007)

Other Nursing Concerns:

  • Anxiety management
  • Improving hearing and communication
  • Promote wound healing and return of skin integrity
  • Patient and significant other’s knowledge about the disease, treatment, and prevention.
  • Initiating self-care and continuing care at home

Medications

Anti-infective: Ofloxacin Otic

An antibiotic of Fluoroquinolone class, which inhibits bacterial DNA gyrase and prevents DNA replication in susceptible bacteria. Kills susceptible to aerobic gram-positive and gram-negative organisms. Indication and dosage: for Chronic Suppurative Otitis Media –  twice daily on the affected ear for 14 days.

Anti-infective :Amoxicillin Oral

An antibiotic of Aminopenicillin type that inhibits cell- wall synthesis during bacterial multiplication. Kills susceptible bacteria. Indication and dosage: for severe infections of the ear – 500 mg/capsule  1 capsule every 8 hrs. for 7 days.

Anti-pyretic / Analgesic : Paracetamol

Para-aminophenol derivative, which may produce an analgesic effect by blocking pain impulses, by inhibiting prostaglandin or pain receptor sensitizers. May relieve fever by acting in the hypothalamic heat-regulating center. Relieves pain and reduces fever. Indication and dosage: mild pain or fever – 500 mg/tablet  1 tablet every 4 hrs., PRN.

Pain-reliever: Mefenamic Acid

The analgesic effect of NSAIDs may result from interference with the prostaglandins involved in pain. Prostaglandins appear to sensitize pain receptors to mechanical stimulation or to other chemical mediators. NSAIDs inhibit the synthesis of prostaglandins peripherally and possibly centrally. Like salicylates, NSAIDs exert an anti-inflammatory effect that may result in part from inhibition of prostaglandin synthesis and release during inflammation. Indication and dosage: mild to moderate pain, inflammation, stiffness, swelling, or tenderness caused by trauma or inflammatory process and surgical procedures – Mefenamic Acid 500mg/tablet  1 tablet every 8 hrs. PRN (if there is pain only) on a full stomach.

Medical Management

Local treatment.

Careful suctioning can be done on the affected ear under microscopic guidance performed by an experienced physician. Proper aural toilet habit and ear care. Instillation of antibiotic drops is used to treat purulent discharges. Also, systemic antibiotics may be prescribed as indicated (Smeltzer & Bare, 2004).

Surgical treatment

Surgical procedures include tympanoplasty, ossiculoplasty and mastoidectomy. The extent of damage and infection determines the type of procedure/s to be performed. Tympanoplasty is the surgical reconstruction of the tympanic membrane which requires ossicles reconstruction. This procedure restores middle ear function, close the perforation, prevent recurrent infection and improve hearing. Ossiculoplasty , surgical reconstruction of the middle ear bones to restore hearing by re-establishing the sound conduction mechanism. Mastoidectomy is done to remove the cholesteatoma, gain access to diseased structures, and create a dry and healthy ear (Smeltzer & Bare, 2004; Goldman, 1996).

Complications

Preventing injury from vertigo . Vertigo occurs if the semicircular canals or other areas of the inner ear are traumatized during mastoid surgery (Smeltzer & Bare, 2004).

Preventing altered sensory perception. Facial nerve (cranial nerve VII) injury is a potential but rare complication of mastoid surgery. Chorda tympani nerve disturbance may occur resulting to alteration in taste and dry mouth on the side of the surgery but soon will disappear until the nerve regenerates (Smeltzer & Bare, 2004).

 Health Teaching

Cognitive:  Be able to identify the effects and actions of every medication. Psychomotor:  Patient and significant others will be able to demonstrate the proper administration of drugs (oral and otic). Affective: Be able to understand the importance of adherence to the medication regimen.

Cognitive:  Patient and significant others will be able to understand the procedures in aural toilet habit. Psychomotor:  Patient and significant others will be able to demonstrate the procedures of aural toilet habit. Affective:  Patient and significant others will be able to understand the importance of the procedures of aural toilet habit and the importance of compliance for the early recovery of the patient.

  • Administration of take home medications.a. proper time, dosage and route of administration
  • importance of adherence to medications
  • layman’s explanation on pharmacology and pharmacokinetics of medications
  • does and don’ts when taking the medications
  • proper instillation of otic drops
  • Aseptic techniques in cleaning post-operative site
  • Handwashing techniques

Able to identify that ofloxacin drops and amoxicillin are antibiotics that fight infection and dose should be completed to prevent the recurrence. Identify that paracetamol should only be taken every four hours if there is fever and mefenamic acid if there is pain–both should be taken on a full stomach to prevent GI ulcer. Provide written instructions to memorize the schedule of medication intake. Confidence in performing the proper instillation of otic drops. Understands the importance of full adherence to the medical regimen.

Confidence to perform proper ear cleaning, aural toilet interventions, and instillation of otic drops. The patient and significant others verbalized the importance of handwashing and aseptic techniques when handling the post-operative site. Practice handwashing and applying alcohol most of the time.

Discharge Plan

Medications: Pain-reliever, antipyretic and antibiotic medications as prescribed, with proper instructions on intake, administration, and compliance.

Environmental: Avoidance of noise-pollution and other hearing hazards. Provide sounds soothing to ears.

Treatment: Proper aural toilet habit and aseptic daily post-op wound care with proper instructions on how to perform the procedures.

Health Teaching: Improving hearing and communication techniques. Health promotion and prevention of the possible recurrence and occurrence of complications.

Out-patient: Follow up consultation after 1 week, as indicated, and or as advised. Importance of compliance with continuous care at home.

Diet: Dietary management of foods rich in protein, iron, vitamin C, and zinc. Avoid foods rich in sodium and cholesterol as it may impair tissue healing; sugar control for diabetic clients. Emphasize the importance of diet for rehabilitation and fast recovery.

Health promotion and prevention are the functions of the primary health care system. Mass awareness and education about CSOM can avert social stigma thus, promoting acceptance. This will play a vital role in screening and early detection. A history of at least 2 weeks of persistent ear discharge should alert primary health workers to the problem. Hearing loss can be prevented if people will listen and learn the ways on how it can be avoided.

CSOM may lead to hearing loss or deafness. Antibiotics and surgeries are very expensive. Its effects are not only centered on the social stigma moreover, its burden is on the permanent damage of deafness. But it can be prevented, early detection and management will save thousands of patients from deafness. Early management will save you from the expensive costs of medications, surgery, and hospitalization.

References:

  • Acuin, J. (2004). Chronic Suppurative Otitis Media: Burden of Illness and Management Options. Geneva, Switzerland: World Health Organization Publication, 7–10, 20. Bonin, E. et al. (2003).
  • Adapting Your Practice: Treatment and Recommendations for Homeless Children with Otitis Media. Nashville: Health Care for the Homeless Clinician’s Network, 1–3. Brener T. et al. (2007).
  • Springhouse Nurse’s Drug Guide 2007, 8th ed. Philadelphia: Lippincott Williams & Wilkins, 1175, 934–937, 149–151, 99–101, 75–76. Doenges, M. E. et al. (2008).
  • Nurse’s Pocket Guide: Diagnoses, Prioritized Interventions, and Rationales, 11th ed. Philadelphia: F.A. Davis Company, 409–413, 498–503.
  • Goldman, M. A. (1996). Pocket Guide to the Operating Room, 2nd ed. Philadelphia: F.A. Davis Company, 437–440. Gulanick, M. & Myers, J. L. (2007). Nursing Care Plans: Nursing Diagnosis and Intervention, 6th ed. Singapore: Mosby Elsevier, 108–112, 144–149. Johnson, M. et al. (2007). NANDA, NOC, and NIC Linkages: Nursing Diagnoses, Outcomes, & Interventions, 2nd ed. Singapore: Mosby Elsevier, 557–561, 303–305. McFarland, M. B. & Grant, M. M. (1988).
  • Nursing Implications of Laboratory Tests, 2nd ed. New York: Delmar Publishers, Inc., 19–30. Roth, R. A. (2007). Nutrition & Diet Therapy, 9th ed. Singapore: Delmar and Thomson Asia Pte Ltd., 382. Smeltzer, S.C. & Bare, B.G. (2004).
  • Brunner & Suddarth’s Textbook of Medical-Surgical Nursing, 10th ed. PA: Lippincott Williams & Wilkins, 1802–1805. Sheir, D.N. et al (2006). Hole’s Essentials of Human Anatomy & Physiology, 9th ed. New York: McGraw-Hill Companies, Inc. 

RELATED ARTICLES MORE FROM AUTHOR

Anaphylactic shock case study -the impending doom, ectopic pregnancy case study, pregnacy induced hypertension (pih) case study, gestational diabetes mellitus case study.

Thank you so much mam.. You r doing a gr8 job. Plz show me the path of knowledge..i m a 3rd year b.sc nursing student..uor biadata have inspired a lot.. I want to become like you .. Plzz plzz plzz show me the path.. And help me to become like you..

Sohini chandra : Thank you so much. I believe you can do better than I did. You are still young … and for sure you will be better than me in the near future. Just study hard and focus on your dreams … strive harder beyond what you can do. We always have that adrenaline power… it will be released the more you aspire … have a great day!

LEAVE A REPLY Cancel reply

Save my name, email, and website in this browser for the next time I comment.

Diamonds 5 (Grade 2 AOM, clear fluid)

Treated with antibiotics 3 weeks ago for AOM. Puffy, fussing with ears, restless sleep, poor appetite, URI symptoms. Temperature 39.4.

Left ear, middle ear effusion, not full, no bulging, no erythema, immobile.

Right ear: initially obstructed by cerumen. After cerumen was removed, right tympanic membrane same as left.

Questions for discussion with faculty

  • How would you treat this patient?
  • The mother would like to know how she might remove wax at home – what would you tell her?
  • Acute Otitis Media More
  • Development Team More
  • Tympanic Membrane Anatomy More
  • Diagnostic Criteria More
  • Patient Evaluation More
  • Therapy & Management Guidelines More
  • Case 1 More
  • Case 2 More
  • Case 3 More
  • Case 4 More
  • Case 5 More
  • Case 6 More
  • Case 7 More
  • Case 8 More
  • Case 9 More
  • Faculty Tool Box More
  • Tympanometry More
  • Exercises More
  • Current Literature More
  • Glossary More
  • Resources for Caregivers More

U.S. flag

An official website of the United States government

The .gov means it’s official. Federal government websites often end in .gov or .mil. Before sharing sensitive information, make sure you’re on a federal government site.

The site is secure. The https:// ensures that you are connecting to the official website and that any information you provide is encrypted and transmitted securely.

  • Publications
  • Account settings
  • My Bibliography
  • Collections
  • Citation manager

Save citation to file

Email citation, add to collections.

  • Create a new collection
  • Add to an existing collection

Add to My Bibliography

Your saved search, create a file for external citation management software, your rss feed.

  • Search in PubMed
  • Search in NLM Catalog
  • Add to Search

Measuring outcomes in nursing centers: otitis media as a sample case

Affiliation.

  • 1 Arizona State University, College of Nursing, Tempe, USA.
  • PMID: 8784879

While it is generally recognized that NPs offer affordable, quality health care, few studies have measured outcomes of clients who seek primary care services from NPs. This pilot study describes the outcomes of children with otitis media who received care from NPs employed in an academic nursing center. Outcome measurements included issues related to timing, level of analysis, and attribution. Parents of 27 children participated in a telephone survey consisting of seven questions relating to the care their children received from NPs and their recovery path. Although every respondent reported having a positive visit at the nursing center, concerns for NPs surfaced during the process of measuring outcomes. This study emphasizes the need for measuring outcomes in nursing clinics and demonstrates one way to measures client outcomes, revealing both general health care and specific nursing practice implications.

PubMed Disclaimer

Similar articles

  • Characterizing nurse practitioner practice by sampling patient encounters: an APRNet study. Deshefy-Longhi T, Swartz MK, Grey M. Deshefy-Longhi T, et al. J Am Acad Nurse Pract. 2008 May;20(5):281-7. doi: 10.1111/j.1745-7599.2008.00318.x. J Am Acad Nurse Pract. 2008. PMID: 18460169
  • Exploration of nurse practitioner practice with clients who are grieving. White P, Ferszt G. White P, et al. J Am Acad Nurse Pract. 2009 Apr;21(4):231-40. doi: 10.1111/j.1745-7599.2009.00398.x. J Am Acad Nurse Pract. 2009. PMID: 19366382
  • Childhood obesity: prevention practices of nurse practitioners. Larsen L, Mandleco B, Williams M, Tiedeman M. Larsen L, et al. J Am Acad Nurse Pract. 2006 Feb;18(2):70-9. doi: 10.1111/j.1745-7599.2006.00105.x. J Am Acad Nurse Pract. 2006. PMID: 16460413
  • Evidence-based practice for nurse practitioners with clinical pathways. Dykes PC, Wheeler K. Dykes PC, et al. Clin Excell Nurse Pract. 1999 Sep;3(5):291-7. Clin Excell Nurse Pract. 1999. PMID: 10763628 Review.
  • Health-related quality of life, satisfaction, and economic outcome measures in studies of prostate cancer screening and treatment, 1990-2000. McNaughton-Collins M, Walker-Corkery E, Barry MJ. McNaughton-Collins M, et al. J Natl Cancer Inst Monogr. 2004;(33):78-101. doi: 10.1093/jncimonographs/lgh016. J Natl Cancer Inst Monogr. 2004. PMID: 15504921 Review.
  • Search in MeSH

Related information

  • Cited in Books

LinkOut - more resources

Full text sources.

  • Ovid Technologies, Inc.
  • MedlinePlus Health Information
  • Citation Manager

NCBI Literature Resources

MeSH PMC Bookshelf Disclaimer

The PubMed wordmark and PubMed logo are registered trademarks of the U.S. Department of Health and Human Services (HHS). Unauthorized use of these marks is strictly prohibited.

Pardon Our Interruption

As you were browsing something about your browser made us think you were a bot. There are a few reasons this might happen:

  • You've disabled JavaScript in your web browser.
  • You're a power user moving through this website with super-human speed.
  • You've disabled cookies in your web browser.
  • A third-party browser plugin, such as Ghostery or NoScript, is preventing JavaScript from running. Additional information is available in this support article .

To regain access, please make sure that cookies and JavaScript are enabled before reloading the page.

IMAGES

  1. Case Study on Otitis Media #otitismedia @NursingCriteria

    nursing case study on otitis media

  2. Otitis Media [+ Free Cheat Sheet]

    nursing case study on otitis media

  3. Otitis Media Nursing Care Planning and Management: Study Guide

    nursing case study on otitis media

  4. NCP on otitis media

    nursing case study on otitis media

  5. Otitis Media Nursing Care Planning and Management: Study Guide

    nursing case study on otitis media

  6. Otitis Media Nursing Diagnosis & Care Plan

    nursing case study on otitis media

VIDEO

  1. Otitis media || types || etiology|| clinical features|| treatment|| For medical students| hindi ||

  2. Case Study / Case Presentation On Otitis Media || OTITIS MEDIA || B.sc. Nursing || ANN ||

  3. #Case study on Tuberculosis🫁

  4. otitis media ( ear disorder ) msn-2

  5. #nursing #notes #OTITIS MEDIA #study #bscnursing

  6. otitis media case presentation / case study .#gnm #bscnursing #mscnursing #nursing #nursingstudent

COMMENTS

  1. Otitis Media Nursing Care Planning and Management: Study Guide

    Otitis media can lead to various symptoms, including ear pain, fever, hearing difficulties, and fluid buildup behind the eardrum. This article aims to serve as a comprehensive nursing guide to otitis media, diving into its causes, clinical manifestations, diagnostic methods, medical management, and nursing interventions.

  2. Evidence-Based Case Reviews: Acute otitis media

    In a prospective Finnish cohort study, earache was about 7 times more likely to be elicited from a child with AOM than from a child who did not have AOM (LR, 7.3). 26 However, a more important finding is that 40% (48/121) of the children with AOM in this study had no apparent earache (LR for no earache, 0.4).

  3. Acute Otitis Media

    Acute otitis media is defined as an infection of the middle ear space. It is a spectrum of diseases that includes acute otitis media (AOM), chronic suppurative otitis media (CSOM), and otitis media with effusion (OME). Acute otitis media is the second most common pediatric diagnosis in the emergency department, following upper respiratory infections. Although otitis media can occur at any age ...

  4. Nursing Care Plan (NCP) for Otitis Media / Acute Otitis Media (AOM)

    By the end of this lesson, nursing students will be proficient in developing a Nursing Care Plan (NCP) for Otitis Media, specifically Acute Otitis Media (AOM). Gain a comprehensive understanding of the pathophysiology and etiology of Otitis Media, with a focus on Acute Otitis Media (AOM). Differentiate between AOM and other types of ear infections.

  5. Etiology, Diagnosis, Complications, and Management of Acute Otitis

    Introduction and background. Acute otitis media (AOM) is defined as "the rapid onset of signs and symptoms of inflammation in the middle ear" [].]. Recurrent otitis media occurs when episodes of AOM are repeated on three separate and well-documented occasions in a period of the last six months or four or more occasions in the last 12 months [].A more complicated presentation of otitis media is ...

  6. Otitis Media Nursing Diagnosis & Care Plan

    Rationale: Otitis media that is caused by a bacterial infection may need a full course of antibiotics. Inform the parents or guardian to speak slowly and clearer to the child. Encourage them to face the child while speaking. Rationale: To promote good communication between the child and the caregiver.

  7. Otitis Media: Rapid Evidence Review

    Otitis Media: Rapid Evidence Review

  8. Acute Otitis Media

    Read chapter 2 of Infectious Diseases: A Case Study Approach online now, exclusively on AccessPharmacy. AccessPharmacy is a subscription-based resource from McGraw Hill that features trusted pharmacy content from the best minds in the field. ... Acute Otitis Media. In: Cho JC. Cho J.C.(Ed.), Ed. Jonathan C. Cho. eds. Infectious Diseases: A Case ...

  9. Treatment of Acute Otitis Media in Children under 2 Years of Age

    To be eligible for enrollment in the study, children were required to have received at least two doses of pneumococcal conjugate vaccine and to have acute otitis media that was diagnosed on the ...

  10. Case Challenge: Acute Otitis Media in Children

    Case Challenge: Acute Otitis Media in Children--Best Management Strategies. Gordon H. Sun, MD, MS. Disclosures. March 28, 2018. ... Acute Otitis Media in Young Children Today; 2001.

  11. Case 13-1999

    Less than 0.1 percent of cases of otitis media and mastoiditis are caused by M. tuberculosis. 22 In a study reported in 1986, tuberculous otitis media in adults was associated with M. tuberculosis ...

  12. PDF Acute Otitis Media (English)

    Acute otitis media is extremely common in children. In fact, it is one of the most common diagnosis in children who are seen in outpatient settings, and is one of the most common reasons for antibiotic therapy. The peak incidence of AOM is between 6 months and 2 years of age. Three out of four children will experience at least one ear infection ...

  13. Acute otitis media case study

    Acute otitis media case study. Case Study 1: HPI: A mother presents with her 2-year-old African American male child with complaints of ear pain and decreased sleep. Earlier this week, he had a runny nose and congestion with a mild cough that occurred mostly when lying down. His temperature is 100.7. PE: Pt is walking around room, exploring in ...

  14. Otitis Media Case Study

    NUR 418: Parent/Child Nursing Care. Garcia Nogueiras. Name: Claudia Garcia Nogueiras Date 10/24/ Case Study: Acute Otitis Media. Instructions: Read chapter 17 page 578-584 and complete the case study on otitis media. The. case study must be typed and the file uploaded on Canvas This assignment will account for 2% of. the Active Learning Exercises (ALEs) course grade.

  15. Otitis Media and Tonsillitis

    CASE STUDY PROGRESS. The primary care provider (PCP) diagnoses C. with bilateral otitis media and strep pharyngitis. C. is given a prescription for Augmentin 600 mg bid PO × 7 days. she is to be discharged to home with instruc- tions to follow-up with the ear, nose, and throat (eNT) specialist. 5.

  16. Case Study: Otitis Media

    Case Background. A middle ear infection, or otitis media, is most often of bacterial origin and commonly follows an upper respiratory infection. The bacteria usually enter the middle ear via the surface of the auditory tube mucus membrane. Inflammation of the tissues in the middle ear results from the infection.

  17. Chronic Suppurative Otitis Media Nursing Case Study

    Chronic Suppurative Otitis Media Case Study. Chronic suppurative otitis media (CSOM) is caused by recurrent ear infections resulting in tympanic membrane perforation characterized by persistent foul-smelling otorrhea from the middle ear. Misconception on ear infection as related to poor hygiene and embarrassing foul-smelling ear discharges ...

  18. Case 2

    Case 2. Child age 8 months. Treated with antibiotics 3 weeks ago for AOM. Puffy, fussing with ears, restless sleep, poor appetite, URI symptoms. Temperature 39.4. Left ear, middle ear effusion, not full, no bulging, no erythema, immobile. Right ear: initially obstructed by cerumen. After cerumen was removed, right tympanic membrane same as left.

  19. Measuring outcomes in nursing centers: otitis media as a sample case

    This pilot study describes the outcomes of children with otitis media who received care from NPs employed in an academic nursing center. Outcome measurements included issues related to timing, level of analysis, and attribution. Parents of 27 children participated in a telephone survey consisting of seven questions relating to the care their ...

  20. Otitis Media Case Study-2

    NUR 418 FCC: Parent Child Nursing. Name_____ Case Study Date_____ Acute Otitis Media. Instructions: Read chapter 17 page 578-584 and complete the case study on otitis media. The. case study must be typed and the file uploaded on Canvas This assignment will account for 2% of. the Active Learning Exercises (ALEs) course grade.

  21. N154 peds Case Study 3

    Nursing care Monitor and record vital signs closely. Encourage and assist the parent to hold and comfort the client. ... N154 Pediatrics Case Study Case study 3: acute otitis media. Please describe the pathology, risk factor, clinical manifestations, labs, nursing intervention, patient educations and treatment/management.

  22. Understanding Antibacterial Agents: Case Study Analysis

    NCM 106 - PHARMACOLOGY CASE STUDY: CLIENT REQUIRING AN ANTIBACTERIAL AGENT DIRECTION: Read and analyze the case. Answer the accompanying questions. FHB, who is 6 years old, has otitis media. The physician ordered Amoxicillin 250 mg every 8 hours. The nurse asks FHB's mother if FHB is allergic to any drugs, and her mother says she is allergic to penicillin.